You are on page 1of 42

PRAYAS - 2022

Test – 24
Economy
Topics covered:
❖ Taxation and Public finance in India
❖ Important International Economic Origination
❖ Important Indian Economic bodies
❖ Reports, Index
❖ Poverty & Unemployment
❖ Important Key terminologies in news

e
in
nl
l.o

PRAYAS-2022
ia
er
at
cm
ps
.u
w
w
w
Contact us :info@onlyias.com

OnlyIAS Nothing Else Visit :dpp.onlyias.in


Contact : +91-7007 931 912

Q.1) With reference to taxation in India, consider the presents a preliminary outcome budget to the
following statements: Ministry of Finance
1. Indirect taxes have a deflationary impact. Q.5) One of the major objectives of introducing the
2. Direct taxes have an inflationary impact. Goods and Services Tax is to remove the cascading
Which of the given statement(s) is/are correct? effect of taxes. How does the Goods and Services Tax
a) 1 only (GST) remove the cascading effect that was present
b) 2 only in the earlier tax regimes?
c) Both 1 and 2 1. By subsuming multiple taxes such as central
d) Neither 1 nor 2 excise, customs duty, central sales tax and value
added tax
Q.2) With reference to the public finance, consider 2. By introducing the mechanism of input tax credits
the following statements: Select the correct answer by using the codes given
1. All revenues received by the government by way below:
of taxes whether direct or indirect except the a) 1 only
loans raised through T-Bills (treasury bills) are b) 2 only
credited to the consolidated fund of India c) Both 1 and 2
2. Receipts from treasury bills are included in the d) Neither 1 nor 2
Public Account of India as treasury bills are short
term instruments. Q.6) Under which of the following circumstances
Which of the given statement(s) is/are incorrect? may the Securities Transaction Tax arise?
a) 1 only 1. Profits are made on a certain security
b) 2 only 2. Company pays dividend to the owners
c) Both 1 and 2 3. A security is purchased or sold
d) Neither 1 nor 2 Select the correct answer using the codes given
below:
Q.3) If the amount authorized to be expended for a a) 1 only
particular service for the current financial year is b) 1 and 3 only
found to be insufficient for the purpose of that year c) 3 only
or when a need for new service has arisen during the d) 1, 2 and 3
current financial year, then which of the following is
laid before the parliament? Q.7) Which one of the following statements
a) Demand for Excess Grants regarding the levying, collecting and distribution of
b) Appropriation bill for grants from Contingency Land Revenue is correct?
Fund a) The Union levies, collects and distributes the
c) Supplementary Demand for Grants proceeds of land revenue between itself and the
d) None of the above states
b) The Union levies, and states collect it on behalf of
Q.4) Which of the following is not a feature of the the Union and all the proceeds of land revenue
Outcome Budget? are distributed among the states
e

a) Outcome budgeting makes government c) The Union levies and collects the land revenue tax
in

programmes more outlay oriented in which but all the proceeds are distributed among the
nl

outcomes of programmes are measured in terms states


l.o

of Rupees d) The states levy, collect and retain the proceeds of


b) It indicates the actual physical performance in the land revenue tax with themselves
ria

previous year, current year and targeted


e

performance during the projected (next) year. Q.8) With reference to the Resource building
at

c) It is a progress card on what various Ministries and mobilization of the IMF, consider the following
cm

and Departments have done with the outlays in statements:


the previous annual budget 1. Resources for IMF loans to its members on non-
ps

d) Outcome Budgeting was first introduced in 2005- concessional terms are provided by member
06, since then, every ministry handling a sector countries, primarily through their payment of
.u

quotas.
w
w

PRAYAS TEST 24 1
w
Contact us :info@onlyias.com

OnlyIAS Nothing Else Visit :dpp.onlyias.in


Contact : +91-7007 931 912

2. Multilateral and bilateral borrowing serve as a 5. The Global Report. E. UNISDR


second and third line of defense, respectively, by (United Nations Office for Disaster Risk
providing a temporary supplement to quota Reduction).
resources. Select the correct answer from the codes given
3. The quotas decreased whereas the multilateral below :
borrowings increased due to the global pandemic a) 1- B, 2- A, 3- C, 4- E, 5- D.
situation. b) 1- B, 2- C, 3- A, 4- D, 5- E.
Which of the statements given above is/are true ? c) 1- C, 2- B, 3- A, 4- E, 5- D.
a) 1 and 2 only d) 1- C, 2- D, 3- C, 4- A, 5- E.
b) 2 and 3 only
c) 3 only Q.12) Which among the following reports is not
d) 1, 2 and 3. published by the World Economic forum ?
a) Global Information Technology Report.
Q.9) Consider the following statements with respect b) Global Financial Stability Report.
to SARFAESI Act. c) Travel and Tourism Competitiveness Report.
1. The provisions of the SARFAESI Act are not d) Global Competitiveness Report.
applicable to the Cooperative Banks.
2. The minimum loan size for debt recovery under Q.13) Which of the following organizations brings
this act is 20 lakhs rupees. out the publication known as ‘ World Investment
Which of the statements given above is/are true? Report ‘ ?
a) 1 only a) BIS (Bank of International Settlement).
b) 2 only b) UNCTAD (United Nations Conference on Trade
c) Both 1 and 2 and Development).
d) Neither 1 nor 2 c) WB (World Bank).
d) IMF (International Monetary Fund).
Q.10) Which among the following statements
regarding the Asset Reconstruction company is Q.14) Consider the following statements with respect
incorrect ? to the National Investment Infrastructure Fund.
a) The first asset reconstruction company (ARC) of 1. Registered as a company under Companies Act,
India was set up under SARFAESI Act. 2013.
b) The asset reconstruction companies or ARCs are 2. It will solely invest in green field projects.
registered under the RBI. 3. It is India’s first sovereign wealth fund.
c) ARCs can buy all kinds of debts from banks and 4. The Governing Council of the fund consists of only
financial institutions so that the latter can clean Government shareholders with a CEO.
up their balance sheets. Select the correct answer using the codes given
d) ARCs in India shall be set up by state-owned and below :
private sector banks, as proposed in the recent a) 1 and 2 only
Union Finance Budget. b) 2, 3 and 4 only
c) 1 and 3 only
e

Q.11) Match the following lists with respect to the d) 1, 2, 3 and 4


in

publications and the organisations that publish it.


nl

List- I ( Reports). Listi- II (Organisations). Q.15) With reference to World Happiness Report
l.o

1. World Development Report. A. International 2021, consider the following statements :


Energy Agency. 1. It is released by the UN Sustainable Development
ria

2. Southeast Asia Energy Outlook. B. IBRD (World Solutions Network


e

Bank.) 2. India has improved slightly in ranking compared to


at

3. Industrial Development Report. C. UN Industrial previous year ranking as well as outperformed


cm

Development Organization. neighbouring country Pakistan and China


4. Global Assessment Report. D. UNHCR 3. As per the report, Women as the head of the
ps

(United Nations High Commissioner for Refugees). government was one of the factor supporting
successful COVID-19 Strategies
.u

Which of the above statements is/are correct ?


w
w

PRAYAS TEST 24 2
w
Contact us :info@onlyias.com

OnlyIAS Nothing Else Visit :dpp.onlyias.in


Contact : +91-7007 931 912

a) 1 & 2 only a) It is Published annually by International Monetary


b) 2 & 3 only Fund(IMF)
c) 1 & 3 only b) Growth rate for India in 2021 is stronger than that
d) 1, 2 & 3 of China.
c) Report has suggested to include Green
Q.16) With reference to the National Coal Index Investment as priority
rolled out by Ministry of Coal, consider the following d) China was the only major economy to have a
statements : positive rate of 2.3% in 2020
1. It is a quality index which reflects on the grade of
coal put on the commercial auction of coal mines Q.20) With reference to Economic Freedom Index
2. National Coal Index(NCI) includes grades of coking 2021, Consider the following statements :
coal only 1. It is Published annually by US think tank , The
Which of the following statements is/are correct ? Heritage Foundation
a) 1 only 2. Indian Economy remains in the mostly unfree
b) 2 only category as per the Index
c) Both 1 & 2 3. Rule of Law is among one of the categories
d) Neither 1 nor 2 included to determine the Index
Which of the following statements is/are correct ?
Q.17) With reference to the Inclusive Internet Index a) 1 & 2 only
2021, Which of the given statements is Incorrect ? b) 3 only
a) The 2021 report is released by Economic c) 2 & 3 only
Intelligence Unit (EIU) in Partnership with d) 1, 2 & 3
Facebook
b) The Report seeks to measure the extent of Q.21) With reference to India Energy Outlook 2021,
accessibility as well as availability of Internet consider the following statements
c) Literacy and cultural acceptance are one of the 1. It is released by United Nations sustainable
criteria used to decide score in the Index Development Solutions Network
d) According to the 2021 Index, People in Upper 2. India has accounted for about 10 percent of world
middle income and higher income countries relied growth in PPP(purchasing power parity) in last
more on online education during the COVID-19 three decades
pandemic 3. India's share in the growth in renewable energy is
the second-largest in the world, after China.
Q.18) The 'State of Finance for Nature' Report has Which of the following statements is/are correct ?
been released jointly by which of the following a) 1 & 2 only
institution/s : b) 2 & 3 only
1. World Bank c) 1 & 3 only
2. World Economic Forum d) 1, 2 & 3
3. United Nations Economic and Social Council
(ECOSOC) Q.22) Poverty estimation in India is carried out by ;
e

4. United Nations Environment Programme a) Ministry of Rural Development and Ministry of


in

5. Economics of Land Degradation Housing and Urban Poverty Alleviation at their


nl

6. Economic Intelligence Unit respective demarcated rural and urban areas.


l.o

Which of the following is/are correct ? b) Office of the Registrar General and Census
a) 1, 3 & 6 only Commissioner, India under Ministry of Home
ria

b) 2, 4 & 5 only Affairs.


e

c) 4, 5 & 6 only c) Niti Aayog based on the data collected by the


at

d) 1, 3 & 6 only National Sample Survey Office under the Ministry


cm

of Statistics and Programme Implementation


Q.19) With reference to the World Economic Outlook (MOSPI).
ps

report released recently, Which of the following d) Ministry of Social Justice and empowerment in
statements is incorrect with respect to the outlines collaboration with respective state Governments.
.u

and different aspects of the report ?


w
w

PRAYAS TEST 24 3
w
Contact us :info@onlyias.com

OnlyIAS Nothing Else Visit :dpp.onlyias.in


Contact : +91-7007 931 912

Q.23) What can be the possible reason for a) Poverty is generally caused by a sudden crisis or
determining poverty on the basis of consumption loss and is often temporary.
expenditure instead of income ? b) Poverty that persists because of internal
a) All people consume but not all earn. psychological factors driven by financial issues.
b) Presence of domination of the unorganized sector c) Poverty that is caused by a violent and unhealthy
in India. environment because of overcrowding.
c) Hindrance in collecting income data. d) Poverty due to less job opportunities, less access
d) All of the above to services, less support for disabilities and quality
education opportunities.
Q.24) Which of the following items is/are present in Q.28) A substantial percentage of Indian population
NSSO’s Mixed Recall Period for measuring lives in slums than it does in China. This refers to an
consumption? example of what kind of poverty ?
1. Food items a) Absolute poverty
2. Clothing b) Relative poverty
3. Foot wear c) Urban poverty
4. Institutional medical expenses d) All of the above.
5. Education
Select the correct answer from the options given Q.29) Recently Tholpavakkoothu was seen in the
below. news, is related to:
a) 1, 4 and 5 a) Martial Art
b) 1 only b) Bhimbetka painting
c) 2, 3, 4 and 5 only c) Rock cut paintings
d) 1, 2, 3, 4 and 5 d) Shadow puppetry

Q.25) With reference to Poverty Gap Ratio, consider Q.30) Recently,' The KURMA App' was frequently
the following statements; seen in news related to -
1. The poverty gap ratio is the mean shortfall of the a) To give information on government schemes and
total population from the poverty line (counting programs on one platform.
the non-poor as having zero shortfall), expressed b) Provide information on newly discovered flora
as a percentage of the poverty line. and fauna.
2. There was a nearly 50% decline in PGR both in c) For turtle conservation.
rural and urban areas during 2004-05 to 2011-12, d) None The Above
reflecting that the conditions of the poor have
improved both in rural and urban areas. Q.31) Binjarpuri, Motu, Ghumusari, recently seen in
Which of the statements given above is/are correct? news, refer to breeds of -
a) 1 only a) Cattle
b) 2 only b) Buffalo
c) Both 1 and 2 c) Sheep
d) Neither 1 nor 2 d) Goat
e
in

Q.26) Which of the following options is not true, with Q.32) ‘Inspiration4’ recently seen in the news refers
nl

respect to Head Count Ratio? to:


l.o

a) It is widely used and represents the cut-off point a) It is the world's first all-civilian mission to orbit.
below which people are considered as poor. b) It is a new scalable genetic control system that
ria

b) The Head Count ratio does give insights about the uses a CRISPR-based approach to suppress
e

severity of poverty. populations.


at

c) Below the Poverty Line category is marked with c) It is the first resource mapping mission on any
cm

the help of its value. other celestial body.


d) None of the above. d) It is a scheme to enhance the number of PhDs in
ps

Electronics System Design & Manufacturing


Q.27) Situational Poverty can be defined as; (ESDM) and IT/IT Enabled Services (IT/ITES)
.u

sectors in the country


w
w

PRAYAS TEST 24 4
w
Contact us :info@onlyias.com

OnlyIAS Nothing Else Visit :dpp.onlyias.in


Contact : +91-7007 931 912

Q.33) REX MKII recently seen in the news is - Q.38) Consider the following factors:
a) a remote-controlled armed robot. 1. Fear of currency war.
b) a long-range patrol aircraft. 2. Dear Money policy of USA’s federal reserves.
c) a short-range, ground-to-air, air defence system. 3. Stiff competition by stock markets around the
d) a mobile, surface-to-air missile system. world.
4. Rise in crude oil prices.
Q.34) Athikaraya, Kayame, Neelam samba recently Which of the above factors are responsible for the
seen in news refers to: fall in India’s forex reserves?
a) Higher biomass yielding bamboo species a) 1, 2, and 3 only
b) Folk varieties of rice b) 1, 3, and 4 only
c) Small rat-like animals that lived around 500 years c) 3 and 4 only
ago d) 1, 2, 3, and 4 only
d) A species of cricket found in Karnataka
Q.39) In relation to the Exchange rate system
Q.35) ‘XENON1T’, recently seen in the news, refers followed by India, which of the statements given
to? below is/are correct?
a) It is a Gamma Irradiation technology for the shelf 1. India follows a dual exchange rate system
life of food products. consisting of the official and the market rates.
b) It is a virus strain that causes haemorrhagic fever. 2. The official exchange rate of India has a direct
c) It is an experiment to detect dark matter and dark bearing on the market exchange rate.
energy. Select the correct answer using the codes given
d) It is the world’s largest galaxy detected through below:
the Hubble Space Telescope. a) 1 only
b) 2 only
Q.36) Consider the following statements regarding c) Both 1 and 2
Special Drawing Rights (SDR) of IMF: d) Neither 1 nor 2
1. The value of SDRs are immune from daily
exchange rate fluctuations. Q.40) Consider the following statements:
2. The least weightage is accorded to the latest 1. The appreciation and depreciation of a currency is
currency added to the SDR basket. a feature of the floating exchange rate regime.
Which of the statements given above is/are correct? 2. The official annual rate of appreciation and
a) 1 only depreciation of an asset are fixed by a country.
b) 2 only 3. The concept of deficit and surplus exists only in
c) Both 1 and 2 relation to the Current Account.
d) Neither 1 nor 2 Which of the statements given above is/are correct?
a) 1 and 2 only
Q.37) Consider the following statements: b) 2 and 3 only
1. The Blue Box subsidies are not trade distorting c) 1 and 3 only
e

while the green box subsidies are specially d) 1, 2, and 3


in

designed to minimize trade distortions.


nl

2. The per farmer Amber Box subsidy in India is less Q.41) In relation to the Extended Fund Facility of the
l.o

as compared to the same given in the developed IMF, which of the statements given below is most
countries. appropriate?
ria

3. The De-minimis entitlement of the subsidy under a) It is the additional money that can be raised by a
e

the Amber Box applies to both the developed and member country from the IMF and is pegged at
at

developing countries. five times its SDR quota.


cm

Which of the statements given above is/are correct? b) It is a form of loan raised amongst the member
a) 1 and 2 only countries with each other without any collateral.
ps

b) 2 and 3 only c) It refers to any amount of money raised by a


c) 1 and 3 only member country to deal with the BoP crisis
.u

d) 1, 2, and 3 subject to the fulfilment of certain pre-conditions.


w
w

PRAYAS TEST 24 5
w
Contact us :info@onlyias.com

OnlyIAS Nothing Else Visit :dpp.onlyias.in


Contact : +91-7007 931 912

d) It refers to interest free loans raised from the IMF 2. Invest India’s NIRVIC Scheme aims at boasting
to deal with the BoP crisis on the fulfillment of exports and increasing investment in India.
certain pre-conditions. Which of the statements given above is/are correct?
a) 1 only
Q.42) Consider the following pairs: b) 2 only
Currency Feature c) Both 1 and 2
1. Hard currency Least liquidity d) Neither 1 nor 2
2. Soft currency Easily available
3. Heated Currency Currency under Q.46) Consider the following statements on the
depreciation impact of global trade in the light of COVID
4. Cheap Currency Currency with the public pandemic:
Which of the statements given above is/are correctly 1. The contraction in trade and GDP was more
matched? severe for the Advanced Economies than for the
a) 1, 2, and 3 only Emerging Markets and Developing Economies.
b) 2, 3, and 4 only 2. Corporate defaults on loans coupled by widening
c) 1, 3, and 4 only fiscal deficit have increased the vulnerability for
d) 1, 2, and 4 only the global economy.
Which of the statements given above is/are correct?
Q.43) Consider the following statements regarding a) 1 only
SEZ (Special Economic Zones): b) 2 only
1. They can be created by the Centre and the state c) Both 1 and 2
governments only. d) Neither 1 nor 2
2. The creation of SEZs is not permitted in the
agricultural sector. Q.47) Consider the following statements regarding
3. There cannot be a common use of the commercial the pharmaceutical sector of India:
infrastructure between a SEZ and a non SEZ 1. India’s pharmaceutical industry is third largest in
entity. the world in terms of volume and value.
Which of the statements given above is/are 2. Drug formulations and biologicals are the second
incorrect? largest exported commodity in the Merchandise
a) 1 and 2 only exports.
b) 2 and 3 only Which of the statements given above is/are correct?
c) 1 and 3 only a) 1 only
d) 1, 2, and 3 b) 2 only
c) Both 1 and 2
Q.44) Consider the following statements regarding E- d) Neither 1 nor 2
commerce:
1. A 100% FDI is permitted in the market-place Q.48) Consider the following statements regarding
based E-commerce models. trade agreements:
2. In a marketplace-based model, the seller and 1. The Comprehensive Economic Partnership
e

platform owner are separate entities. Agreement is an example of a common market


in

3. The recent e-commerce rules, 2020 are applicable with common currency and tariffs.
nl

only to the market-place based operators. 2. In a Custom union, members apply a common
l.o

Which of the statements given above is/are correct? external tariff for non-member countries.
a) 1 and 2 only 3. The Comprehensive and Progressive Agreement
ria

b) 2 and 3 only for Trans-Pacific Partnership (CPTPP) is the revised


e

c) 1 and 3 only TPP after the withdrawal of the USA.


at

d) 1, 2, and 3 Which of the statements given above is/are correct?


cm

a) 1 and 2 only
Q.45) Consider the following statements: b) 2 and 3 only
ps

1. Cabinet Committee on Economic Affairs approves c) 1 and 3 only


FDI from the countries sharing borders with India. d) 1, 2, and 3
.u
w
w

PRAYAS TEST 24 6
w
Contact us :info@onlyias.com

OnlyIAS Nothing Else Visit :dpp.onlyias.in


Contact : +91-7007 931 912

Q.49) Consider the following statements: c) Both 1 and 2


1. The membership BIMSTEC is composed of all d) Neither 1 nor 2
SAARC countries excluding Pakistan.
2. Mekong Ganga cooperation is an agreement Q.53) Consider the following statements:
between India and all countries through which 1. Data Free flow with Trust is an initiative of G 20
Mekong flows. countries.
Which of the statements given above is/are correct? 2. JAI comprising Japan, Australia, and India was
a) 1 only formed on the sidelines of Buenos Aires summit of
b) 2 only G 20.
c) Both 1 and 2 3. Regional Anti- terrorist Structure is an important
d) Neither 1 nor 2 component of the SCO.
Which of the statements given above is/are correct?
Q.50) In the context of D 10 countries, which of the a) 1 and 2 only
statements given below is/are correct? b) 2 and 3 only
1. It is a USA proposed grouping to foster free trade c) 1 and 3 only
with the Asian countries. d) 1, 2, and 3
2. It is a group of three democratic countries along
with the G7. Q.54) Maastricht Treaty and the Digital Single
3. One of its aims is to surpass China for the supply Market Strategy often seen in news are related to?
of 5G technologies. a) European Commission
4. Australia, India, and Japan are the three b) Eurozone
democratic countries part of it outside G7 c) European Union
Select the correct answer using the codes given d) Schengen Area
below:
a) 1 and 2 only Q.55) Consider the following statements:
b) 2 and 3 only 1. Exports from the countries under the Generalized
c) 3 and 4 only System of Preferences List are subjected to lower
d) 1 and 4 only import tariffs on all commodities.
2. Global system of Trade preferences is a trade
Q.51) Consider the following groupings: agreement among the developed nations of the
1. Asian Pacific Economic Cooperation. world.
2. Indian Ocean Rim Association. Which of the statements given above is/are correct?
3. Organization for Economic Co-operation and a) 1 only
Development. b) 2 only
4. South Asian Free Trade Area. c) Both 1 and 2
Which of the above groupings does not include India d) Neither 1 nor 2
as its member?
a) 3 only Q.56) In context of public finance in India, consider
b) 1, 2, and 3 only the following:
e

c) 1 and 3 only 1. Securities issued against Small savings funds


in

d) 2 and 4 only 2. Market Stabilization Scheme Bonds


nl

3. Securities issued to International Monetary Fund


l.o

Q.52) In the context of the Cairns grouping, which of (IMF)


the statements given below is/are incorrect? Which of these is/are component(s) of internal debt?
ria

1. It is a group of agricultural exporting nations a) 1 only


e

demanding liberalization of agricultural trade. b) 1 and 2 only


at

2. India is a founding member of the group owing to c) 2 only


cm

its maximum population employed in the sector. d) 1, 2 and 3


Select the correct answer using the codes given
ps

below: Q.57) The government's deficit excluding the interest


a) 1 only payment on the previous debt is
.u

b) 2 only a) Primary Deficit


w
w

PRAYAS TEST 24 7
w
Contact us :info@onlyias.com

OnlyIAS Nothing Else Visit :dpp.onlyias.in


Contact : +91-7007 931 912

b) Budget Deficit d) In a buyback company repurchases a certain


c) Fiscal Deficit amount of its outstanding shares and extinguishes
d) Revenue Deficit them once taken back

Q.58) Consider the following: Q.62) Consider the following:


1. Revenue Deficit 1. Fiscal Deficit
2. Capital Expenditure 2. Revenue Deficit
3. Effective Revenue Deficit 3. Primary Deficit
Grants or expenditure used for creation of Assets are Which of the following deficit(s) focuses only on the
calculated under which of the given heads? present fiscal imbalances?
a) 1 and 3 only a) 1 and 2 only
b) 2 only b) 1 only
c) 2 and 3 only c) 2 and 3 only
d) 3 only d) 3 only
Q.59) In context of the Public Finance in India,
consider the following statements: Q.63) Recently, the G7 countries signed an accord for
1. A large share of revenue deficit in the fiscal deficit the Global Minimum Corporate Tax. It will help in
indicates that a large part of borrowing is being resolving which of the following issues?
used for investment a) Transfer Pricing of the Multinational Corporations
2. Revenue deficit in the economy cannot be b) Investments will flow equally to low income
eliminated countries
Which of the given statement(s) is/are incorrect? c) Reduce corporate corruption in India
a) 1 only d) Reduce Base erosion and profit shifting of MNCs
b) 2 only
c) Both 1 and 2 Q.64) Consider the following with reference to Indian
d) Neither 1 nor 2 Tax System:
1. Corporation Tax
Q.60) Consider the following statements about the 2. Dividends Distribution Tax
recently formed Consumer Welfare Fund: 3. Minimum Alternative Tax
1. The Consumer Welfare Fund was set up under the 4. Stamp Duty
Consumer Protection Act, 2019 Which of the following is/are direct tax(es)
2. The fund has been set up by the Department of applicable to private sector companies in India?
Consumer Affairs (Ministry of Consumer Affairs, a) 1 and 2 only
Food & Public Distribution). b) 2, 3 and 4 only
Which of the given statement(s) is/are correct? c) 1 and 3 only
a) 1 only d) 1, 2 and 3 only
b) 2 only
c) Both 1 and 2 Q.65) Which of the following is not one of the four
d) Neither 1 nor 2 critical reforms, on the implementation of which the
e

states will be allowed to additionally borrow over


in

Q.61) Recently, the government has proposed to their current liabilities?


nl

introduce a Buy Back Tax on companies, which of the a) Water and Sanitation Reforms
l.o

following is incorrect about the buyback of shares? b) Power Sector Reforms


a) Companies resort to the buyback of shares to c) Ease of doing business Reforms
ria

escape the capital gains tax d) One Nation One Ration Card
e

b) Buyback helps in improving the earnings per share


at

for continuing shareholders and perks up the Q.66) Consider the following transactions:
cm

return on equity 1. Investment in Mutual Funds


c) It can also be done when the promoters want to 2. Buying an immovable property which is under
ps

hike their stake in the company, sometimes to construction


avoid any takeover threats 3. Transfer or shares and debentures
.u
w
w

PRAYAS TEST 24 8
w
Contact us :info@onlyias.com

OnlyIAS Nothing Else Visit :dpp.onlyias.in


Contact : +91-7007 931 912

The stamp duty will be applicable on which of the d) 1 and 3 only


given transaction(s)?
a) 1 and 2 only Q.70) Consider the following statements with respect
b) 2 only to the Fiscal Responsibility and Budget Management
c) 1, 2 and 3 Act, 2003:
d) None of the above 1. The Comptroller and Auditor General of India shall
review, on a half-yearly basis, the trends in
Q.67) Consider the following statements with receipts and expenditure in relation to the budget
reference to the Composition Levy: and place before both Houses of Parliament the
1. Composition Levy is an alternative method of levy outcome of such reviews
of direct tax designed for small businesses whose 2. According to the Budget 2021-22, the government
turnover is up to Rs. 1.5 crore is planning to continue on the path of fiscal
2. The small businesses can avail to input tax credits consolidation, achieving a fiscal deficit level below
after the payment of the composition levy 4.5% of GDP by 2025-2026
Which of the given statement(s) is/are incorrect? Which of the given statement(s) is/are correct?
a) 1 only a) 1 only
b) 2 only b) 2 only
c) Both 1 and 2 c) Both 1 and 2
d) Neither 1 nor 2 d) Neither 1 nor 2

Q.68) Consider the following statements with Q.71) Which of the following document(s) is/are
reference to the Goods and Services Tax published while presenting the Union Budget?
Compensation Cess: 1. Fiscal Policy Strategy Statement
1. To compensate states, a new tax, offsetting the 2. Medium-Term expenditure Framework
revenue losses and proceeds from which will be Statement
transferred to the states has been introduced 3. Annual Financial Statement
2. In the present design of GST, there is no provision 4. Demand for Grants
for revenue compensation for the Union 5. Supplementary Demand for grants
government. Select the correct answer using the codes given
Which of the given statement(s) is/are correct? below:
a) 1 only a) 1, 3, 4 and 5 only
b) 2 only b) 1, 2, 3 and 5 only
c) Both 1 and 2 c) 1, 2, 3 and 4 only
d) Neither 1 nor 2 d) 1, 3 and 4 only

Q.69) Which of the given statements is not correct Q.72) Consider the following:
regarding the tax system of India? 1. Agricultural implements
1. According to the Article 265, no tax should be 2. Silkworm laying cocoon
levied/collected except the authority of the law 3. Processed fish
e

2. When the government reduces taxes during the 4. Raw silk


in

recession period, it is said to be following a pro- 5. Electricity


nl

cyclical fiscal policy Which of the following good(s) is/are exempted from
l.o

3. The Goods and Services tax(GST) is a consumption the application of the Goods and Services Tax?
and destination based indirect tax a) 1, 4 and 5 only
ria

4. GST being a value added tax, it's collection will b) 2, 4 and 5 only
e

increase with the increase in the Nominal Gross c) 2, 3 and 4 only


at

Domestic Product (GDP) d) 1, 3 and 4 only


cm

Select the correct answer using the codes given


below: Q.73) Which of the following statements is not
ps

a) 1 and 2 only correct with respect to off Budget or extra budgetary


b) 2 only borrowings?
.u

c) 2 and 4 only
w
w

PRAYAS TEST 24 9
w
Contact us :info@onlyias.com

OnlyIAS Nothing Else Visit :dpp.onlyias.in


Contact : +91-7007 931 912

a) Off budget borrowings are not added to the fiscal 1. Equalization levy has been introduced under the
deficit Income Tax Act, 1961 to tax foreign companies
b) These borrowings are not a part of total which have a significant local base
government debt, however the interest payments 2. It is a direct tax levied on the profits made by the
on these are paid through the Consolidated fund companies like Google, Amazon
of India 3. Equalization levy is applicable to foreign
c) Outstanding extra-budgetary liabilities need to be companies which have a tangible presence in
clearly identified and eliminated in a time-bound India
manner as per the amended FRBM Act of 2018 Which of the following statement(s)
d) Off-budget borrowings are loans that are taken is/are incorrect?
not by the Centre directly, but by another public a) 1 only
institution which borrows on the directions of the b) 2 and 3 only
central government c) 3 only
d) 1, 2 and 3
Q.74) Consider the following statements:
1. Most of India's external debt is short term in Q.78) The term 'escape clause' is often seen in news
nature in context of
2. Commercial borrowings form the largest a) Related to base erosion and profit shifting
component of India's external debt b) Related to interest subvention scheme for
Which of the statement(s) given above is /are exporters
correct? c) Related to climate action plan passed in Paris
a) 1 only agreement
b) 2 only d) Related to fiscal responsibility and budget
c) Both 1 and 2 management act
d) Neither 1 nor 2
Q.79) With reference to the Fugitive Economic
Q.75) Which one of the following Offenders Act, 2018, consider the following
committees/commission has recommended the statements.
establishment of an independent fiscal council? 1. A Fugitive Economic Offender is a person against
a) Narsimhan committee whom an arrest warrant has been issued for
b) Abid Hussain Committee committing any offence listed in the Act, and the
c) XIV Finance Commission value of the offence is at least Rs 1000 crore.
d) XV Finance Commission 2. The Department of Economic Affairs in
collaboration with the Ministry of Finance may
Q.76) Consider the following statements about the declare a person as a Fugitive Economic Offender.
debt sustainability of the Indian economy: 3. There is provision of confiscation of properties of
1. As far as the economy is growing at a higher rate a Fugitive Economic Offender which are proceeds
the difference between the interest rate on debt of crime, Benami properties and any other
and growth rate will be negative in most of the properties both in India and abroad.
e

cases Which of the statements given above is/are correct ?


in

2. Debt is sustainable if the interest rate on debt is a) 1 and 2 only


nl

more than the growth rate of the economy b) 2 and 3 only


l.o

Which of the given statement(s) is/are correct? c) 3 only


a) 1 only d) 1, 2 and 3
ria

b) 2 only
e

c) Both 1 and 2 Q.80) With reference to The Bilateral Netting of


at

d) Neither 1 nor 2 Qualified Financial Contracts Act, 2020, consider the


cm

following statements.
Q.77) Consider the following statements with 1. Bilateral netting is a legally enforceable
ps

reference to the equalization levy: arrangement between a bank and a counterparty


that creates a single legal obligation covering all
.u

included individual contracts.


w
w

PRAYAS TEST 24 10
w
Contact us :info@onlyias.com

OnlyIAS Nothing Else Visit :dpp.onlyias.in


Contact : +91-7007 931 912

2. The provisions of the bill will apply to Qualified d) ‘Achieving Nutritional Security in India: Vision
Financial Contracts between two qualified 2030 ' report was recently released by it.
financial market participants where at least one
party is an entity regulated by the specified Q.84) Which of the following schemes are financed
authorities RBI, SEBI, IRDAI, PFRDA or the IFSCA. by NABARD?
Which of the statements given above is/are correct ? 1. Paramparagat Krishi Vikas Yojana.
a) 1 only 2. Dairy Entrepreneurship Development Scheme
b) 2only 3. Commercial production units of organic inputs
c) Both 1 and 2 scheme
d) Neither 1 nor 2 4. Interest Subvention Scheme
5. National Livestock Mission.
Q.81) In the context of Stock Exchanges, the term Select the correct answer from the codes given
‘Demutualisation‘ refers to as, below:
a) Transfer of shares from the brokers to the public. a) 1, 2, 3 and 4 only
b) Sudden fall of prices of the commodities. b) 1 and 2 only
c) Process of cooperative approach in managing the c) 2, 3, 4 and 5 only
shares. d) 1, 2, 3, 4 and 5.
d) Trade executed in the share market that is settled
without the approval of a party in question. Q.85) Consider the following statements about
Goods and Service Tax.
Q.82) Which among the following statements is 1. The GST Bill was first introduced in 2014 as The
incorrectly mentioned with respect to the Securities Constitution (122nd Amendment) Bill.
and Exchange Board of India ? 2. The Union Finance Minister is the Chairperson of
a) It can draft regulations, conduct inquiries, pass the GST council that is formed by the President of
rulings and impose penalties. India.
b) Autonomous organization accountable to the 3. Recent amendments in the GST Act
Parliament of India. made generating e-Invoices mandatory for
c) Securities Appellate Tribunal (SAT) constituted to companies having turnover of more than 50 crore.
protect the interest of entities that feel aggrieved Select the correct answer using the codes given
by SEBI’s decision can also hear and dispose of below:
appeals against orders passed by the Insurance a) 1 and 2 only
Regulatory Development Authority of India b) 1 only
(IRDAI) under the Insurance act, 1938. c) 2 only
d) Chairman of SEBI is nominated by the Union d) 1, 2 and 3
Finance Ministry with consultation with the RBI.
Q.86) Which of the following food items does not
Q.83) Which among the following statements is come under the ambit of Goods and Service tax (GST)
incorrectly mentioned relating to the provisions of ?
NABARD ? 1. Milk in Tetra Pack
e

a) Committee to Review the Arrangements For 2. Cereals


in

Institutional Credit for Agriculture and Rural 3. Packed Paratha


nl

Development (CRAFICARD) under the 4. Tender coconut


l.o

chairmanship of B. Sivaraman recommended the 5. Plain roti/chapati


formation of NABARD. 6. Frozen fish products
ria

b) Non-financial interventions, innovations, 7. Tea leaves.


e

technology and institutional development for Select the correct answer using the codes given
at

securing prosperity are among the main below :


cm

objectives of NABARD. a) 1, 2, 4, 5 and 7 only


c) Facilitate credit flow for promotion and b) 2, 4 and 7 only
ps

development of agriculture and agricultural c) 3, 4, 5 and 6 only


related industry only. d) 3, 5 and 7 only.
.u
w
w

PRAYAS TEST 24 11
w
Contact us :info@onlyias.com

OnlyIAS Nothing Else Visit :dpp.onlyias.in


Contact : +91-7007 931 912

Q.87) Which of the following are the functions of the 4. Composite water management Index.
Reserve Bank of India ? 5. Sustainable Tourism in the Himalayan Region.
1. The RBI shall recommend the denominational 6. DigitalPayments Trends, Issues and Opportunities.
values of the notes as well as the discontinuance Select the correct answer from the codes given
of issue of notes. below:
2. RBI sells the foreign currency in the foreign a) 1, 2 ,3 and 5 only
exchange market when its supply decreases in the b) 3, 4 .5 and 6 only
economy and vice-versa. c) 1, 2, 4, 5 and 6 only
3. Setting of Inflation targeting. d) 1, 2, 3, 4, 5 and 6.
4. Maintaining price stability in the economy.
Select the correct answer answer using the codes Q.91) With reference to the recent proposed
given below : amendments made to the Consumer Protection (E-
a) 1, 2 and 3 only commerce) Rules 2020, consider the following
b) 2 and 3 only statements.
c) 1, 2 and 4 only 1. Conventional e-commerce flash sales are banned.
d) 1 and 4 only 2. A filter mechanism to identify goods based on
country of origin has to be incorporated by the e-
Q.88) Which of the following matters of the commerce platforms.
Commercial Banks are regulated by the Reserve Bank 3. Mandatory requirement for registration number
of India ? issued by Department for Promotion of Industry
1. Controlling credit creation by commercial banks. and Internal Trade.
2. Merger and winding up of commercial banks. Which of the statements given above is/are correct ?
3. Appointment of the board and its chairman of a a) 1 and 2 only
commercial bank. b) 1 only
Select the correct answer using the codes given c) 2 and 3 only
below : d) 1, 2 and 3
a) 1 only
b) 1 and 2 only Q.92) ‘New Arrangement to Borrow (NAB)’, is related
c) 3 only to which of the following organisations?
d) 1, 2 and 3. a) International Monetary Fund
b) World Bank
Q.89) Which of the following provisions is/are c) NITI Aayog
amended in the present Companies Act, then that of d) New Development Bank.
Companies Act of 1956 ?
1. Shareholders permitted for a private company Q.93) Which of the following statements is
increased. incorrectly mentioned with respect to Asian
2. A single person can start a company. Development Bank?
3. Decreasing the limit on maximum partners. a) China holds the largest share in Asian
4. Duties of the director of a company is defined. Development Bank followed by India.
e

Select the correct answer using the codes given b) To eradicate extreme poverty is one of the main
in

below : goals.
nl

a) 1, 2 and 4 only c) Resolution passed by the United Nations


l.o

b) 3 only Economic Commission for Asia and the Far East in


c) 2, 3 and 4 only 1963 resulted in the formation of Asian
ria

d) 1, 2,3 and 4 Development Bank.


e

d) Board of Governors is the highest policy making


at

Q.90) Which among the following index/reports are body of Asian Development Bank and consists of
cm

published by the NITI Aayog ? members from outside Asia region also.
1. Health Index of India.
ps

2. Agricultural Marketing and Farmer Friendly Q.94) With reference to Asian Infrastructure and
Reform Index. Investment Bank, consider the following statements:
.u

3. State rooftop solar attractiveness Index.


w
w

PRAYAS TEST 24 12
w
Contact us :info@onlyias.com

OnlyIAS Nothing Else Visit :dpp.onlyias.in


Contact : +91-7007 931 912

1. The bank allows non-sovereign entities to apply


for membership. Q.99) Poverty trap can be defined as;
2. It enjoys a Permanent Observer Status in the a) a mechanism that forces people to remain poor.
United Nation. b) a situation where people working informally,
Which of the above given statements is/are correct? without proper job contracts and thus sans any
a) 1 only legal protection
b) 2 only c) fudging with data relating to poverty to gain
c) Both 1 and 2 cheap and long term credit.
d) Neither 1 nor 2 d) criteria for the World Bank to invest in projects to
invest in underdeveloped countries.
Q.95) Which of the following statements is correct
about the New Development Bank? Q.100) With reference to the Gini Coefficient,
a) During the sixth BRICS Summit in Fortaleza (2014), consider the following statements;
the BRICS leaders signed the Agreement 1. A value of 0 indicates perfect equality (where
establishing the New Development Bank (NDB). everybody has the same wealth/income) and 1
b) New Development Bank assigns votes based on indicates perfect inequality (that is, where one
capital share. person owns all the wealth in a country).
c) As per Fortaleza Declaration, the New 2. The Gini Coefficient is given by the World Bank.
Development Bank Africa Regional Center shall be Which of the above given statements is/are correct ?
established in Brazil concurrently with the a) 1 only
headquarters in Shanghai. b) 2 only
d) Recently it confirmed financing for the ‘Assam c) Both 1 and 2
Intra-State Transmission System Enhancement d) Neither 1 nor 2
Project’, for building power supply infrastructure
in Indian state of Assam.

Q.96) Which of the following organizations in India


publishes the Housing Price Index ?
a) Housing Development Finance Corporation
b) NITI Aayog
c) National Housing Bank
d) Reserve bank of India.

Q.97) Which of the following statements is incorrect


with respect to the International Bank for
Reconstruction and Development (IBRD) ?
a) It is the lending arm of the World Bank Group and
offers loans to middle-income developing
countries.
e

b) The bank can finance any projects that increase


in

economic activity and are responsible for


nl

eliminating poverty.
l.o

c) India has been the single largest borrower of the


Bank.
ria

d) IBRD has maintained a triple-A rating since 1959.


e
at

Q.98) International Poverty Line’ is a concept given


cm

and set by which of the following organizations?


a) World Bank
ps

b) International Monetary Fund


c) United Nations Development Programme
.u

d) International Labour Organization.


w
w

PRAYAS TEST 24 13
w
Contact us :info@onlyias.com

OnlyIAS Nothing Else Visit :dpp.onlyias.in


Contact : +91-7007 931 912

Q.2) Ans: c
Exp:
o The Accounts of the government of India are kept
in three parts: Consolidated Fund of India,
Contingency Fund and Public Accounts of India.
• Statement 1 is Incorrect: All revenues received
by the government by way of taxes whether
direct or indirect and other receipts flowing to
the government in connection with the conduct
of government business like receipts from
Railways, Post, transport, government PSU's etc.
are credited into the Consolidated Fund of
India. Similarly, all loans raised by the government
by issue of public notifications, treasury bills and
loans obtained from foreign governments and
international institutions are credited into this
fund.
• Statement 2 is Incorrect: Treasury bills are
also debt instruments through which the
government raises loans, hence they are also
credited to the Consolidated Fund of
India. Moreover, all expenditures incurred by the
government for the conduct of its business
including repayment of internal and external debt
and release of loans to States/ Union Territory
governments for various purposes are debited
against this fund and no amount can be
withdrawn from the Fund without the
authorization from the Parliament.

Q.1) Ans: d Q.3) Ans: c


Exp: Exp:
o Taxes are of two types: Direct and Indirect Option C is the Correct answer.
Taxes. As the name itself suggests, direct tax is • In case the amount authorized to be expended for
paid directly to the government while the indirect a particular service for the current financial year is
tax is paid indirectly. found to be insufficient for the purpose of that
• Statement 1 is Incorrect: Indirect taxes are year or when a need has arisen during the current
basically taxes that can be passed on to another financial year for supplementary or additional
e

entity or individual. They are usually imposed on expenditure upon some 'new service' not
in

a manufacturer or supplier who then passes on contemplated in the budget for that year then
nl

the tax to the consumer. Thus, they do not look at the President causes to be laid before both the
l.o

the consumer’s ability to pay but are the same for Houses of Parliament another statement showing
everyone who buys the goods or services. the estimated amount of that expenditure which
ria

Hence, indirect taxes are usually inflationary. is called "Supplementary Demand for Grants".
• Statement 2 is Incorrect: Direct taxes, on the one
e

• The "Supplementary Demands for Grants" are


at

hand, are taken from an presented to and passed by the House before the
cm

individual’s earnings, while indirect taxes are end of the financial year.
imposed on goods that consumers buy.
ps

Furthermore, direct taxes are calculated based on Q.4) Ans: a


the paying capacity of the individual. Hence, Exp:
.u

direct taxes are not inflationary at all. Option A is the Correct answer.
w
w

PRAYAS TEST 24 14
w
Contact us :info@onlyias.com

OnlyIAS Nothing Else Visit :dpp.onlyias.in


Contact : +91-7007 931 912

• Outcome Budgeting was first introduced in 2005- consumer bears the burden of the multiple taxes
06 by the then finance minister, P Chidambaram. imposed on every stage of production. Such a
It is practiced by most of the Ministries while situation leads to inflationary prices. Thus, ‘Tax
preparing their budget details and submitting it to on tax’ plagued the indirect tax structure of India
the Ministry of Finance for the preparation of the prior to GST.
annual budget towards the end of February. It • Statement 1 is Incorrect: GST removed the
replaced the system of performance budgeting cascading effect by subsuming 17 taxes.
by ministries, introduced in 1969 following the However, the customs duty still continues on
recommendations of the Administrative Reforms imports whereas, the exports come under the
Commission to address its weaknesses such as category of ‘Zero rated’ goods and services. Here
lack of a clear relationship between the financial are a few taxes that will be subsumed in GST:
and performance budgets and inadequate target 1. Central sales tax
setting for the ensuing year. 2. Central sales tax
• Outcome based budgeting is a practice of 3. Service tax
suggesting and listing estimated outcomes of 4. Octroi
each programme or schemes designed. 5. Special additional duty of customs
• Under outcome budgeting, the document 6. Additional excise duty
shows physical dimensions of the financial 7. Countervailing duty
budget indicating the actual physical performance 8. Purchase tax
in the previous year, current year and targeted 9. Entertainment tax
performance during the projected (next) year. 10. Central Excise duty
Outcome Budget helps in: 11. State Value Added Tax
1. Better service delivery • Statement 2 is Correct: Input tax credit means
2. Decision making that when a manufacturer pays the tax on his
3. Evaluating programme performance and results output, he can deduct the tax he previously paid
4. Improving programme effectiveness on the input he purchased. Here, while paying the
5. Make budgets cost effective tax on his output, he can deduct or take credit for
6. Accountable resource management the tax he paid while purchasing inputs. Thus, it is
7. Outcome budgeting makes the government the input tax credit that makes the GST efficient
programmes more result oriented, instead of and constructive and also eliminates the
outlay oriented. cascading effect. Providing ITC is a big
Difference between Outlay, Outcome and Outputs: administrative and technological work under the
1. Outlays are financial resources deployed for GST platform.
achieving certain outcomes.
2. Outputs are a measure of the physical quantity of Q.6) Ans: c
the goods or services produced through a Exp:
government scheme or programme. They are • The Securities Transaction Tax is a tax levied at
usually an intermediate stage between ‘outlays’ the time of purchase and sale of securities listed
and ‘outcomes. For example, construction of a on stock exchanges.
e

health care centre is the ‘output’, while increase • Statement 1 is Incorrect: When profit is gained
in

in the life expectancy is the ‘final outcome’ or on a certain security, the gains are taxed as per
nl

‘impact’. the Capital Gains Tax. The initiative behind


l.o

3. Outcomes or impact are the end results of various introducing STT was to curb evading of capital
government initiatives. Going beyond mere gains tax on profits earned by transecting in
ria

‘outputs’, they cover the quality and effectiveness securities.


e

of the goods or services. • Statement 2 is Incorrect: When a company pays


at

profits to the owners, the sum of money is


cm

Q.5) Ans: b charged through the Dividend Distribution


Exp: Tax and not the securities transaction tax.
ps

• Cascading tax effect is also termed as “tax on • Statement 3 is Correct: The Securities Transaction
tax”. This effect occurs when a good is taxed on Tax is a tax levied at the time of purchase and
.u

every stage of production. As a result, the final sale of securities listed on stock exchanges. This
w
w

PRAYAS TEST 24 15
w
Contact us :info@onlyias.com

OnlyIAS Nothing Else Visit :dpp.onlyias.in


Contact : +91-7007 931 912

tax was introduced in the 2004 Union Budget and among members. The most recent increase in
came into effect from 1 October 2004. The rate of quotas, to SDR 477 billion (US$ 651 billion), was
STT differs based on the type of security traded agreed under the 14th Review (concluded in
and whether the transaction is a purchase or a December 2010, effective from January 2016.)
sale. The 15th Review was concluded in February 2020
without a quota increase. In
Q.7) Ans: d its resolution concluding the 15th Review, the
Exp: Board of Governors also provided guidance on the
Option D is the Correct answer. 16th Review, expected to be concluded no later
• Taxes Levied and Collected and Retained by than December 15, 2023.
States: • In January 2021, a reform of the NAB took effect
Some taxes and duties exclusively belong to following consents from NAB participants,
states. They are mentioned in the State list. Every almost doubling the size of the NAB to SDR 361
state is entitled to levy, collect and appropriate these billion (US$521 billion) for the period from 2021
taxes. The taxes are: to 2025.
1. Duty on succession to agricultural land
2. Estate duty on agricultural land Q.9) Ans: b
3. Land revenue Exp:
4. Tax on agricultural income • Statement 1 is incorrect: A five-judge Constitution
5. Tax on land and buildings Bench of the Supreme Court (SC) has ruled that all
6. Capitation taxes co-operative banks in the country could make use
7. Tax on mineral rights of the SARFAESI Act to make recovery against
8. Tax on the consumption or sale of electricity defaulting persons.
9. Tax on vehicles • Cooperative banks come under the category of
10. Tax on the sales and purchase of goods (other banks as defined under Section 2(1)(c) of the
than newspaper) for e.g. Sales tax Sarfaesi Act, and the recovery procedures
11. Tolls mentioned under that law apply to cooperative
12. Tax on professions, trades and employment banks as well.
• Statement 2 is correct: "To improve credit
Q.8) Ans: a discipline while continuing to protect the interest
Exp: of small borrowers, for NBFCs with minimum
• Statement 1 is correct: Quotas are the IMF’s main asset size of Rs 100 crores, the minimum loan size
source of financing. Each member of the IMF is eligible for debt recovery under the Securitisation
assigned a quota, based broadly on its relative and Reconstruction of Financial Assets and
position in the world economy. Enforcement of Security Interest (SARFAESI) Act,
• Statement 2 is correct: The New Arrangements to 2002 is proposed to be reduced from the existing
Borrow (NAB) constitutes a second line of defense level of Rs 50 lakh to Rs 20 lakh," the finance
to supplement IMF resources to forestall or cope minister said.
with an impairment of the international monetary
e

system. Through the NAB, a number of member Q.10) Ans: c


in

countries and institutions stand ready to lend Exp:


nl

additional resources to the IMF. • Option c is incorrect: The ARC can take over only
l.o

• Bilateral borrowing Agreements serve as a third secured debts which have been classified as a
line of defense after quotas and the NAB.Since the non-performing asset (NPA). Assets.The ARCs take
ria

onset of the global financing crisis, the IMF has over a portion of the debts of the bank that
e

entered into several rounds of bilateral borrowing qualify to be recognised as Non-Performing


at

agreements (BBAs) to ensure that it can meet the Assets.


cm

financing needs of its members. BBAs serve as a • All the rights that were held by the lender (the
third line of defense after quotas and the NAB. bank) in respect of the debt would be transferred
ps

• Statement 3 is incorrect: The IMF regularly to the ARC. Funds required to purchase such
conducts general reviews of quotas to assess the
.u

debts can be raised from Qualified Buyers. Banks,


adequacy of overall quotas and their distribution Financial Institutions, Insurance companies, State
w
w

PRAYAS TEST 24 16
w
Contact us :info@onlyias.com

OnlyIAS Nothing Else Visit :dpp.onlyias.in


Contact : +91-7007 931 912

Financial Corporations, State Industrial imbalances that are highlighted in one of the
Development Corporations, trustee or ARCs IMF's other publications, the World Economic
registered under SARFAESI and Asset Outlook topics covered in the GFSR usually
Management Companies registered under SEBI include systemic risk assessments in worldwide
that invest on behalf of mutual funds, pension financial markets, worldwide debt management,
funds, FIIs, etc are qualified buyers. emerging economic markets and current
• Option a, b and d are correct. The very first Asset economic crises that could affect finances
Reconstruction Company (India) Ltd or Arcil, was worldwide.
set up in 2002 by -four banks: SBI, ICICI Bank, PNB • Global Information Technology Report: A report
and IDBI Bank under SARFAESI Act. The asset produced in cooperation with INSEAD, World
reconstruction companies or ARCs are registered Economic Forum, and the World Bank, the Global
under the RBI. The ARC proposed in the Budget Information Technology Report ranks the world
will be set up by state-owned and private sector economies in terms of networked readiness and
banks, and there will be no equity contribution effects on economic growth and productivity.
from the Centre. • Travel and Tourism Competitiveness
Report: Published every two years by the World
Q.11) Ans: a Economic Forum, the Travel & Tourism
Exp: Competitiveness Report and Index compares the
• Following are some of the reports/indices along competitiveness of 140 economies and measures
with the organization from where it is published. the set of factors and policies that enable the
1. Ease Of Doing Business: IBRD (World Bank). sustainable development of the Travel & Tourism
2. World Development Report: IBRD (World Bank). (T&T) sector, which in turn contributes to the
3. World Energy Outlook: International Energy development and competitiveness of a Country.
agency. • Global Competitiveness Report: The Global
4. Southeast Asia Energy Outlook: International Competitiveness Report (GCR) is a yearly report
Energy Agency. published by the World Economic Forum. Since
5. World Oil Outlook: OPEC ( Organization of the 2004, the report ranks countries based on the
Petroleum Exporting Countries). Global Competitiveness Index.The report
6. The Global Report: UNHCR (United Nations High "assesses the ability of countries to provide high
Commissioner for Refugees). levels of prosperity to their citizens". This in turn
7. Global Assessment Report: UNISDR (United depends on how productively a country uses
Nations Office for Disaster Risk Reduction). available resources. Therefore, the Global
8. Industrial Development Report: UN Industrial Competitiveness Index measures the set of
Development Organization. institutions, policies, and factors that set the
9. Global Informationation Technology Report: sustainable current and medium-term levels of
World Economic Forum. economic prosperity
10. World Happiness Report: Sustainable
Development Solutions Network. Q.13) Ans: b
Exp:
e

Q.12) Ans: b • Option b is correct: The ‘ World Investment


in

Exp: Report ‘ focuses on trends in foreign direct


nl

• Option b is correct: The Global Financial Stability investment (FDI) worldwide, at the regional and
l.o

Report (GFSR) is a semiannual report by country levels and emerging measures to improve
the International Monetary Fund (IMF) that its contribution to development.
ria

assesses the stability of global financial markets • Established in 1964 as a permanent


e

and emerging-market financing. It is released intergovernmental body, UNCTAD is the principal


at

twice per year, in April and October. The GFSR organ of the United Nations General Assembly
cm

focuses on current conditions, especially financial dealing with trade, investment and development
and structural imbalances, that could risk an upset issues. It is also the United Nations focal point for
ps

in global financial stability and access to financing the least developed countries. Major functions
by emerging-market countries. It emphasizes the include maximizing the trade, investment and
.u

ramifications of financial and economic development opportunities of developing


w
w

PRAYAS TEST 24 17
w
Contact us :info@onlyias.com

OnlyIAS Nothing Else Visit :dpp.onlyias.in


Contact : +91-7007 931 912

countries and assist them in their efforts to been operating since 2012 under the auspices of
integrate into the world economy. the UN Secretary-General. SDSN mobilizes global
scientific and technological expertise to promote
Q.14) Ans: c practical solutions for sustainable development,
Exp: including the implementation of the Sustainable
• Statement 1 is correct: The National Investment Development Goals (SDGs) and the Paris Climate
and Infrastructure Fund (NIIF) Limited has been Agreement India showed slight improvement in
incorporated as a company under the Companies ranking since last year's report
Act, 2013, duly authorized to act as investment • Statement 2 is Incorrect : Finding of the Report :
manager of National Investment and Top 5 countries were – Finland (topped the list for
Infrastructure Fund. fourth straight year), Iceland, Denmark,
• Statement 2 is incorrect: It provides funds directly Switzerland, The Netherlands and Sweden.
to commercially viable projects, both Greenfield • India ranked 139th out of the 149 countries in the
and Brownfield, including stalled projects, along list, a slight improvement since last year(ranked
with key transportation projects could be taken 140th).
up initially by the NIIF for funding. • India’s neighbours: Pakistan (105th), Bangladesh
• Greenfield project is one that lacks constraints (101st) and China (84th)
imposed by prior work. Simply means: a new • Statement 3 is Correct : As per the Report,
project. following were the factors supporting successful
• Brownfield project is used to describe land COVID 19 Strategies -:
previously used for industrial purposes or some o Confidence in public institutions: Trusted public
other uses. Brownfield project simply means: an institutions were more likely to choose the right
already existing/ stalled project. strategy and have their populations support the
• Statement 3 is correct: National Investment and required actions.
Infrastructure Fund (NIIF) is India’s first o Income inequality, acting partly as a proxy for
infrastructure specific investment fund or a social trust.
sovereign wealth fund that was set up by the o Learnings from SARS and other earlier pandemics.
Government of India in February 2015. The o Whether the head of the government was a
objective behind creating this fund was to woman
maximize economic impact mainly through
infrastructure investment in commercially viable Q.16) Ans: d
project. Exp:
• Statement 4 is incorrect: There will be a • Statement 1 is Incorrect : Ministry of Coal has
Governing Council of the NIIF which will have started Commercial Auction of coal mines on
Government representatives and experts in revenue share basis using NCI. NCI has been rolled
international finance, eminent economists and out on 4th June 2020. NCI is a price index which
infrastructure professionals. It could include reflects the change of price level of coal in a
representatives from other non-Government particular month relative to the fixed base year.
shareholders. The terms and period of The base year for the NCI is FY 2017-18.
e

appointment of the Governing Council of the NIIF o Prices of coal from all the sales channels of coal,
in

will be as decided by the Government. including import, as existing today are taken into
nl

• Provisions for one or more CEO (depending upon account for compiling the NCI.
l.o

the number of funds created) and a small • Statement 2 is Incorrect : NCI is composed of a
set of 5 sub-indices: 3 for Non-Coking Coal and 2
ria

investment team consisting of limited number of


expert staff, at arm’s length from the for Coking Coal.
e

Government. o As per the grade of coal pertaining to a mine, the


at

appropriate sub-index (Coking and Non-coking) is


cm

Q.15) Ans: c used to arrive at the revenue share.


Exp: o Coking coal is a grade of coal that can be used to
ps

• Statement 1 is Correct : Recently, World produce good-quality coke, one of the key inputs
.u

Happiness Report, 2021 was released by the UN for the production of steel.
Sustainable Development Solutions Network Has
w
w

PRAYAS TEST 24 18
w
Contact us :info@onlyias.com

OnlyIAS Nothing Else Visit :dpp.onlyias.in


Contact : +91-7007 931 912

o Non-coking coals have higher ash content and do • Option b is Correct


not have any caking properties. It is mainly used • The State of Finance for Nature Report has been
as thermal coal for power generation. released. The report recommended more
investments in “Nature-based solutions”.
Q.17) Ans: d • Released by: United Nations Environment
Exp: Programme (UNEP), World Economic Forum
• Option d is Incorrect (WEF) and the Economics of Land Degradation.
• EIU(Economic Intelligence • Purpose: The report tracks the global trends in
Unit) in partnership with Facebook has released t public and private investment in nature-based
he Inclusive Internet Index 2021 solutions. By this, it aims to improve data quality
o Purpose: It seeks to measure the extent of and identify opportunities for governments,
accessibility and affordability of the Internet. Also, businesses and financiers.
the internet should be able to enable positive o Nature-based solutions(NbS) refer to sustainable
social and economic outcomes at the individual management and the use of nature to tackle
and group levels. socio-environmental challenges. These challenges
o Countries Covered: The index assesses the range from disaster risk reduction, climate change
performance of 120 countries representing 98% and biodiversity loss to food and water security as
of global GDP and 96% of the global population. well as human health
o Categories: The Index score is based on the scores • Key Findings of the Report:
of 4 categories; o The current investments in Nature-based
o Availability: It examines the quality and breadth solutions amount to USD 133 billion. This is about
of available infrastructure required for access and 0.10% of global GDP. Public funds make up 86%
levels of Internet usage. and private finance makes up the remaining 14%
o Affordability: It examines the cost of access of these investments.
relative to income and the level of competition in o Public Investment: The largest proportion of
the Internet marketplace. public investment is carried out by the United
o Relevance: It also examines the existence and States, with approximately $36 billion a year in
extent of local language content and relevant NbS spending. It is followed by China, Japan,
content. Germany and Australia.
o Readiness: Furthermore, it examines the capacity o Countries such as Brazil, India and Saudi Arabia
to access the Internet. It includes skills (literacy), are likely spending large amounts of money too.
cultural acceptance, and supporting policy. But they do not report internationally comparable
Literacy measures the level of education and data, according to the report.
preparedness to use the Internet o Private Investment: The private investment in
• Key Findings Related to India: Nature-based solutions is mostly contributed by
o India’s Rank: 49th out of 120 countries in the Commercial financial institutions, Investors
Index. including insurance companies, asset
o India is set to reach one billion internet users by management firms Philanthropies including
2025. There were over 687.6 million internet foundations and endowments
e

users in India in 2020.


in

• Other Findings: Q.19) Ans: a


nl

o Topped by: Sweden ranks 1st in the index, Exp:


l.o

followed by the US and Spain. • Option a is Incorrect


o The majority of countries (77 out of 120) saw • The latest edition of the International Monetary
ria

improvements in internet inclusion overall, in part Fund’s (IMF) World Economic Outlook has raised
e

because of increased availability. its Financial Year (FY) 2021 growth forecast for
at

o People in low and lower-middle-income countries India to 12.5% from 11.5% estimated earlier in
cm

relied more on online education during the January 2021


pandemic in comparison to wealthier countries. o It is a survey by the IMF that is usually published
ps

twice a year in the months of April and October.


Q.18) Ans: b
.u

o It analyzes and predicts global economic


Exp: developments during the near and medium term
w
w

PRAYAS TEST 24 19
w
Contact us :info@onlyias.com

OnlyIAS Nothing Else Visit :dpp.onlyias.in


Contact : +91-7007 931 912

• Key Points of Report • Statement 3 is Correct: The index ranks 12


• Indian economy is expected to grow by 12.5% in indicators from property rights to financial
2021 and 6.9% in 2022. freedom under four categories: rule of law, size
• In 2020, India’s economy witnessed an estimated of government, regulatory efficiency and open
contraction of 8%. markets. The Heritage Foundation's Index of
• Growth rate for India in 2021 is stronger than Economic Freedom is an annual guide published
that of China. to measure the progress made in advancing
• China was the only major economy to have a economic freedom which it claims brings greater
positive growth rate of 2.3% in 2020, and is prosperity. The Foundation also believes that "the
expected to grow by 8.6% in 2021 and 5.6% in ideals of economic freedom are strongly
2022 associated with healthier societies, cleaner
• Suggestions in the Report environments, greater per capita wealth, human
• Health Care -: The emphasis should be on development, democracy, and poverty
escaping the health crisis by prioritising health elimination."
care spending, on vaccinations, treatments, and
health care infrastructure. Fiscal support should Q.21) Ans: b
be well targeted to affected households and firms. Exp:
• Accommodative Monetary Policy -: Monetary • Statement 1 is Incorrect : Recently,
policy should remain accommodative (where the International Energy Agency (IEA) has
inflation is well behaved), while proactively released the India Energy Outlook 2021
addressing financial stability risks using Report which explores the opportunities and
macroprudential tools challenges ahead for India as it seeks to ensure
• Priorities -: The priorities should include green reliable, affordable and sustainable energy for a
infrastructure investment to help mitigate growing population.
climate change, digital infrastructure investment o The India Energy Outlook 2021 is a new special
to boost productive capacity and strengthening report from the IEA’s World Energy Outlook series
social assistance to arrest rising inequality. • Statement 2 is Correct : According to the
report, Industrialisation is a Major Driving Force
Q.20) Ans: d in India's energy sector. Over the last three
Exp: decades, India accounted for about 10% of World
• Statement 1 is Correct: Singapore topped the Growth in Industrial Value-added [in Purchasing
global ranking for the second year in a row in the Power Parity (PPP) terms].
latest Economic Freedom Index published by US o By 2040, India is set to account for almost 20% of
conservative think-tank, The Heritage Global Growth in Industrial value-added, and to
Foundation. lead global growth in industrial final energy
o The creation of the index is consumption, especially in steelmaking
based around economist Adam Smith’s theories i • Statement 3 is Correct : According to the Report :
n ‘The Wealth of Nations’. One of them states o Renewables Energy Resources Demand: India's
that the “basic institutions that protect the liberty share in the growth in renewable energy is the
e

second-largest in the world, after China


in

of individuals to pursue their own economic


interests result in greater prosperity for the larger o Gas Demand: India will become the fastest-
nl

society”. growing market for Natural gas, with demand


l.o

• Statement 2 is Correct: Report commented that: more than tripling by 2040. Natural gas import
dependency increased from 20% in 2010 to
ria

"The Indian economy remains in the mid-range of


the mostly unfree category. A move toward almost 50% in 2019 and is set to grow further to
e

greater economic freedom would require more than 60% in 2040.


at

substantial and broad-based reforms. In addition o Coal Demand: Coal currently dominates India's
cm

to taking steps to improve fiscal health and electricity sector, accounting for over 70% of
strengthen the rule of law, the government would overall generation. Coal demand is seen rising to
ps

need to increase financial freedom and reform the 772 million tonnes in 2040 from the current 590
.u

tax code, the investment regime, and the labour


Q.22) Ans: c
w

code.
w

PRAYAS TEST 24 20
w
Contact us :info@onlyias.com

OnlyIAS Nothing Else Visit :dpp.onlyias.in


Contact : +91-7007 931 912

Exp: 2004-05 to 5.05 in 2011-12 in the urban areas, it


• Poverty is measured based on consumer declined from 6.08 to 2.70 during the same
expenditure surveys of the National Sample period. A nearly 50% decline in PGR both in rural
Survey Organisation. Poverty estimation in India is and urban areas during 2004-05 to 2011-12,
carried out by NITI Aayog’s task force through the reflects that the conditions of the poor have
calculation of poverty line based on the data improved both in urban and rural areas.
captured by the National Sample Survey Office
under the Ministry of Statistics and Programme Q.26) Ans: b
Implementation (MOSPI). Exp:
• Option b is incorrect: The head count ratio (HCR)
Q.23) Ans: d is the population proportion that exists, or lives,
Exp: below the poverty threshold. One of the
• The reason to determine poverty on the basis of undesirable features of the head count ratio is
consumption expenditure instead of income is that it ignores the depth of poverty; if the poor
that dependent people like childrens and senior become poorer, the head count index does not
citizens also consume even though they are not change.
earning. One more reason is even though income
is less people may consume more on the basis of Q.27) Ans: a
their savings or borrowings. The Indian economy Exp:
is dominated by an unorganized sector and it is • Option a is correct: Situational poverty is a period
difficult to collect income data, hence wherein an individual falls below the poverty line
determination of poverty is done on the basis of because of a sudden event. Situational poverty
consumption expenditure instead of income. can be caused by a range of factors, such as: a
divorce, death of the family head, illness, a natural
Q.24) Ans: c disaster or loss of job. These are uncontrollable
Exp: and often unpredictable events that can escalate
• Mixed Recall Period involves estimation of until the person finds themselves without
poverty using consumer expenditure data of 365 material possessions or an income source.
days recall period, for five infrequently purchased
non- food items such as; Q.28) Ans: b
• Clothing Exp:
• Foot wear • Option b is correct: In Relative poverty , the
• Durable goods income/consumption distribution of the
• Education population in different percentile groups is
• Institutional medical expenses, and a 30 day recall estimated and a comparison of the levels of living
period for the remaining items. of the top 5 to 10% with the bottom 5 to 10 % of
o From 1999-2000 onwards, the NSSO switched to the population reflects the relative standard of
the MRP method for estimation of poverty. poverty. Hence a person’s income/consumption
may be above the poverty line but he happens to
e

Q.25) Ans: c be poor in comparison with the person whose


in

Exp: income is above his/hers. This measure is to


nl

• Statement 1 is correct: The Poverty Gap Ratio is calculate inequality in society. In a rapidly
l.o

the gap by which mean consumption of the poor changing world, definitions of poverty based on
relative standards will be constantly changing.
ria

below poverty line falls short of the poverty line.


It indicates the depth of poverty; the more the Poverty is best understood in a relative way –
e

PGR, the worse is the condition of the poor. While what is poor in China is not the same as what is
at

the number of poor people indicates the spread poor in India


cm

of poverty, PGR indicates the depth of the


poverty. Q.29) Ans: d
ps

• Statement 2 is correct : During 2004-05 to 2011- Exp:


Tholpavakoothu:
.u

12, PGR also reduced in both rural and urban


w

areas. While the rural PGR declined from 9.64 in


w

PRAYAS TEST 24 21
w
Contact us :info@onlyias.com

OnlyIAS Nothing Else Visit :dpp.onlyias.in


Contact : +91-7007 931 912

• Tholpavakoothu is a traditional temple art in


Turtle Survival Alliance (TSA):
Kerala having its roots in Palakkad and
• The Turtle Survival Alliance (TSA) was formed in
neighbouring regions. It is a shadow puppetry,
2001 as an International Union for Conservation
also called Nizhalkkuthu and Olakkoothu.
of Nature (IUCN) partnership for sustainable
• It used to be performed in the Bhadrakali temples
captive management of freshwater turtles and
of Palakkad, telling tales from the Ramayana.
tortoises.
Musical Instruments Use- Accompanying
• Initially it was designated a Task Force of the IUCN
instruments include Ezhupara, Chenda and
Tortoise and Freshwater Turtle Specialist Group.
Maddalam.
• It was organized in 2001 in response to the Asian
Turtle Crisis. It has since become a recognized
• The artists have to undergo several years of
force in turtle and tortoise conservation globally.
rigorous training to master this art form. The
• It will help to reduce the rampant and
unsustainable harvest of Asian turtle populations
to supply Chinese markets.
• It aims to ' Zero Turtle Extinctions in the 21st
Century’.

Q.31) Ans: a
Exp:
• Option a is correct: Four breeds of cattle —
puppetry is staged on a special structure Binjharpuri, Motu, Ghumusari and Khariar — and
in temple premises called Koothumadam. two breeds of buffalo — Chilika and Kalahandi
• In Malayalam, Thol means leather, pava means — and one breed of sheep, Kendrapada, have
doll and koothu means play. Though the origin of already received NBAGR recognition.
this ritualistic art form is not known exactly, some • The National Bureau of Animal Genetic
believe it to be as old as 1200 years. Resources (NBAGR) has recognised the Manda
• This entertainment art is performed on a special buffalo as the 19th unique breed of buffaloes
stage called koothumadam in the temple found in India. Manda buffalo are native to
courtyard. It is performed using the mythological Odisha and have ash grey and grey coat colour
figures along with the use of fire and lighting of with copper colour hairs.
lamps behind the screen. The chief puppeteer is • They are found in the Eastern Ghats and plateau
known as ‘Pulavan’. of the Koraput region of Odisha. Indian Council
of Agricultural Research (ICAR) -National Bureau
Q.30) Ans: c of Animal Genetic Resources, Karnal (ICAR-
Exp: NBAGR) is the nodal agency for the registration
The KURMA App: of newly identified germplasm of livestock and
• It is a mobile-based application aimed at turtle poultry of the country.
conservation. It was launched on the occasion of • Its mandate includes Identification, Evaluation,
e

World Turtle Day (23rd May). Characterization, Conservation and sustainable


in

• It is developed by the Indian Turtle Conservation Utilization of Livestock and Poultry Genetic
nl

Action Network (ITCAN) in collaboration with the Resources.


l.o

Turtle Survival Alliance-India and Wildlife


Conservation Society-India.
ria

Q.32) Ans: a
• The KURMA App has a built-in digital field guide Exp:
e

covering 29 species of freshwater turtles and • Option a is correct: Inspiration4 is the world’s
at

tortoises of India, and information on turtle first all-civilian mission to orbit. The flight will be
cm

identification, vernacular names, and threats. privately operated by SpaceX (US Private Space
• It provides users a database to identify a species. Company) using a previously-flown Crew Dragon
ps

capsule launched to low Earth orbit.


.u

o Inspiration4 will blast off from NASA’s Kennedy


Extra Edge by Only IAS
Space Center in Florida on September 15.
w
w

PRAYAS TEST 24 22
w
Contact us :info@onlyias.com

OnlyIAS Nothing Else Visit :dpp.onlyias.in


Contact : +91-7007 931 912

o The mission involves circling the Earth for three o These can supplement the arachidonic acid (ARA)
days, and then splashing down into the Atlantic and docosahexaenoic acid (DHA) in infants
Ocean. Inspiration4 will orbit the Earth at 575km, through breast-feeding.
higher than the International Space Station o DHA and ARA are fatty acids found in breast milk,
(408km) and the Hubble space telescope as well as in some foods, like fish and eggs. These
(547km). This will be the farthest distance rice varieties can be more cost-effective and
travelled by a crewed mission since 2009, when reliable than marketed formula foods.
astronauts last went to repair the Hubble. o In India, many folk varieties like Athikaraya, Dudh-
• Option b is incorrect: precision-guided Sterile sar, Kayame, Neelam samba, Srihati, Maharaji and
Insect Technique(pgSIT) is a new scalable genetic Bhejri are known in folk medicine to enhance milk
control system that uses a CRISPR-based production in lactating women. Other traditional
approach to engineer deployable mosquitoes that varieties like Kelas, DudheBolta and Bhutmoori
can suppress populations. are rich in iron and can be included in diet of
o It alters genes linked to male fertility — creating mothers to treat anaemia
sterile offspring — and female flight in Aedes
aegypti, the mosquito species responsible for Q.35) Ans: c
spreading wide-ranging diseases including dengue • Statement C is correct: XENON1T is a dark matter
fever, chikungunya, and Zika. research project run by the Gran
o pgSIT uses CRISPR to sterilize male mosquitoes Sasso National Laboratory in Italy. This is a deep
and render female mosquitoes, which spread underground research facility which aims
disease, as flightless to detect dark matter particles. It could also be
• Option c is incorrect: VIPER stands for Volatiles used to detect dark energy.
Investigating Polar Exploration Rover. It is a • Other Dark Matter and Energy Experiments:
mobile robot. It is the first resource mapping o LUX-Zeplin
mission on any other celestial body o in US and PandaX-xT project at China
• Option d is incorrect: Union Minister of
Electronics & Information Technology launched Q.36) Ans: d
Phase II of the Visvesvaraya PhD Scheme. It aims Exp:
to enhance the number of PhDs in Electronics • Statement 1 is incorrect: The value of SDR is
System Design & Manufacturing (ESDM) and determined in terms of the US dollars and
IT/IT Enabled Services (IT/ITES) sectors in the is determined on the daily basis based on the daily
country. exchange rate fluctuations. Hence, the value of
SDR undergoes daily fluctuations depending upon
Q.33) Ans: a the exchange rates.
Exp: • Statement 2 is incorrect: The latest currency to be
• Option a is correct: REX MKII is an Israeli remote- added to the SDR basket was Chinese Renminbi.
controlled armed robot that can The other currencies in the basket of SDR includes
patrol the battle zone, track intruders, and the US Dollars, Euros, Japanese Yen, and the
fire. Operated via an electronic tablet, it can be pound sterling. While the maximum weight is
e

equipped with two machine guns, a camera and a assigned to the US Dollars, the minimum
in

sensor. Robots can collect information weightage is to pound sterling. Chinese currency
nl

on ground forces, transport injured soldiers and was the last addition to the basket yet the weight
l.o

supplies in and out of combat, and attack nearby assigned to it is after the dollars and euros.
targets.
ria

Q.37) Ans: b
e

Q.34) Ans: b Exp:


at

Exp: • Statement 1 is incorrect: The WTO’s Agreement


cm

• Option b is correct: Researchers have examined on Agriculture allows a minimum amount of trade
12 folk varieties of Indian rice that can distorting subsidies for both the developed and
ps

supplement the nutritional demand of important developing countries to ensure food security of
fatty acids in undernourished mothers. the country. These subsidies are classified under
.u

three boxes namely, the Green box, the Blue box,


w
w

PRAYAS TEST 24 23
w
Contact us :info@onlyias.com

OnlyIAS Nothing Else Visit :dpp.onlyias.in


Contact : +91-7007 931 912

and the Amber box subsidies. The Green box rate system. In this case, two exchange rates
subsidies do not distort the trade and hence there namely the market exchange rate and the official
is no limit on them. The Blue Box subsidies are rate are announced. The intervention of RBI
also provided without any limits. The subsidies occurs in this system when too much fluctuation is
under the Blue box are allowed by the WTO with witnessed in the exchange rate market.
the condition that they must be designed to • Statement 2 is incorrect: The market variations of
reduce the trade distortion. demand and supply decides the market exchange
• Statement 2 is correct: The per farmer Amber box rate. The official exchange rate is then dependent
subsidy is more for the USA than for India. Even on the market exchange rate and not the other
though India provides for a greater percentage of way round.
subsidies, it is more per farmer for the USA than
for India. Q.40) Ans: c
• Statement 3 is correct: The Amber Box provides Exp:
limitations on the subsidies for both the • Statement 1 is correct: In case
developed and the developing countries. of appreciation, the currency gains its value when
Developed countries are entitled to provide compared to the foreign currency whereas in case
subsidies worth 5% of their production while for of depreciation, the currency loses its value when
the developing countries, the limit is 10%. compared to the foreign currency. Hence,
appreciation and depreciation of a currency can
Q.38) Ans: d occur only when the exchange rate is based on
Exp: market-based fluctuations of the demand and
• Option D is correct: The forex reserves of India is supply. Therefore, it is a feature of the floating
maintained by the Reserve Bank of India and is in exchange rate system.
the form of foreign currencies, gold, Reserve • Statement 2 is incorrect: The concept of
Tranche position of the IMF and the SDR. To appreciation and depreciation are also linked to
strengthen the foreign exchange reserves, there the increase and the decrease in the value of an
needs to be greater incoming of foreign asset. Every asset undergoes wear and tear with
investments in the country which is possible by time and hence depreciates in its value. This
strengthening the investment ecosystem. Fear of depreciation is officially fixed by the
currency war among the major economies like the government annually. However, there is no such
USA and China is likely to reduce the forex official fixation for the appreciation of assets as it
reserves by causing volatility in the exchange rate is for the depreciation of the assets.
and the depreciation of rupee. The dear money • Statement 3 is correct: The deficit and surplus is
policy of the US Federal reserve increases the expressed in terms of Current accounts only and
loan interest rates by the commercial banks and not for the capital account. The current account
hence borrowers provide competitive interest includes the trade in goods and services, along
rates to the investors causing them to be with the transfer payments, and the Income. The
attracted to their domestic market. Huge Balance of Payment crisis occurs when the
competition among the stock exchanges of the country does not have sufficient forex reserves to
e

world causes volatility in the exchange rates and finance its deficit.
in

the investors seek to invest in those markets


nl

which are offering them the best returns. Rising Q.41) Ans: c
l.o

crude oil prices which are brought in dollars leads Exp:


to the more depletion of the foreign exchange of • Option C is correct: Extended Fund Facility of the
ria

the country for the same quantity of import of International Monetary Fund refers to any
e

crude oil and makes our imports costlier. amount of loan or foreign exchange that can be
at

raised by a member country of the IMF to finance


cm

Q.39) Ans: a its Balance of Payment crisis. However, this


Exp: is subjected to certain structural reforms as
ps

• Statement 1 is correct: India has been following suggested by the IMF for the member country.
the floating exchange rate system since 1993. The India is a party to this agreement of the IMF and
.u

Indian model is referred to as the dual exchange during its BoP crisis of 1991, it agreed to the
w
w

PRAYAS TEST 24 24
w
Contact us :info@onlyias.com

OnlyIAS Nothing Else Visit :dpp.onlyias.in


Contact : +91-7007 931 912

structural reforms as suggested by the IMF. These


reforms included the devaluation of the rupee, Q.44) Ans: a
decrease in custom and increase in excise duties, Exp:
reduction of government spending, etc. • Statement 1 is correct: There are two kinds of E-
commerce models: Market-based and Inventory-
Q.42) Ans: b based models. In the market-based model, the
Exp: role of the E-commerce entity is only to provide
• Option B is correct: Hard currency is the most the web platform to connect the sellers and
liquid form of currency. It is short in supply and buyers. It does not own the goods and service and
large in demand in the international market. The merely acts as a facilitator of the transactions
USA's dollar is the strongest currency of the world between the buyer and the seller. 100% FDI is
and is a hard currency. Soft currency is the one permitted in the market-based E-commerce
which is easily available and not so liquid as model. The inventory-based model focuses on the
compared to hard currency. Indian rupee is a soft owning and selling of the goods and services
currency in the International market. The demand directly by the E-commerce entity. There is no FDI
for this currency is not very high. Heated permitted here.
currency is the one that bears the pressure of • Statement 2 is correct: In the market-place
depreciation with the withdrawal of the hard models the sellers and facilitator of sale through a
currency from its market. Huge withdrawal of portal are different entities. This model is
dollars from the Indian market causes the beneficial as it connects the local vendors to the
depreciation of the Indian rupee which is the customers around the country through an online
heated currency in this case. Cheap currency is portal.
the currency with the public. If the government • Statement 3 is incorrect: The recent E-commerce
purchases its securities back from the public, then rules, 2020 are applicable to all the E-commerce
money is infused into the hands of the public entities providing goods and services in
which is the cheap currency. India, whether registered in India or outside India.
These rules seek to strengthen the grievance
Q.43) Ans: d redressal of the customers.
Exp:
• Statement 1 is incorrect: The Special Economic Q.45) Ans: d
Zones or SEZs were created under the provisions Exp:
of the Special Economic Zones or SEZ Act, in • Statement 1 is incorrect: Earlier the Foreign
2005. These are the export hubs of the country Investment Promotion Board was responsible for
and are treated as foreign territory for the approval to the FDI investments in India. After its
purpose of tax and trade related laws. They are abolition, this work is done by the individual
provided with tax holidays and other benefits to sector related ministry in consultation with the
enable them to generate employment and Ministry of Commerce. As per the new norms
infrastructure facilities. They can be created by released by the Government, any FDI coming from
the Central government, the state governments or the countries sharing borders with India would
e

any other private parties. require the approval of the Government of India.
in

• Statement 2 is incorrect: SEZs can be created in The move aimed at curbing China from overtaking
nl

any sector of the economy- Agriculture, businesses in India at a cheap price during the
l.o

Manufacturing and the services. To encourage COVID pandemic.


agro-based food processing industries in the SEZs, • Statement 2 is incorrect: Invest India or
ria

a new category for this purpose was added in the National Investment Promotion and
e

2017 by the Ministry of Commerce and Industry. Facilitation Agency of India is a not-for-profit
at

• Statement 3 is incorrect: To ensure faster growth organization established by the Ministry of


cm

of the SEZs, it has been allowed that social and Commerce and Industry along with the FICCI,
commercial infrastructures located outside the NASSCOM, and others. The Nirvik scheme that
ps

territories of SEZs would be made accessible to aims promotion of exports is a scheme by


them. Hence, the common use of such the Export Credit Guarantee Corporation of
.u

infrastructures have been allowed.


w
w

PRAYAS TEST 24 25
w
Contact us :info@onlyias.com

OnlyIAS Nothing Else Visit :dpp.onlyias.in


Contact : +91-7007 931 912

India, a PSU under the Ministry of Commerce and Trade Agreement which is far more
Industry. comprehensive than a conventional free trade
agreement. In an FTA, the partner countries only
Q.46) Ans: c aim at the reduction of the tariffs. However, if the
Exp: countries go beyond simply reducing the tariffs to
• Statement 1 is correct: As per the Economic relaxing the norms for foreign investment and
Survey 2020-21, the COVID pandemic led to the movement of labor then it forms a
slowdown of the economic activity due the Comprehensive Economic Partnership Agreement.
imposition of lockdowns and the halt of economic It is not a common market that allows free
activity. It was observed that in the Advanced movement of labor along with a common tariff for
Economies, the impact of contraction in GDP and a non-member.
trade volume is much more than that of the • Statement 2 is correct: Customs Union is a Free
Emerging Markets and Developing Economies Trade Agreement where the members of the
(EMDE). The decline in exports and imports was Union charge reduced tariffs to each other while
very steep for the Advanced Economies as they charge a common tariff for external or non-
compared to the EMDEs. member countries.
• Statement 2 is correct: As per International • Statement 3 is correct: Trans-Pacific
Monetary Fund, there is a lot of vulnerability in Partnership or TPP was a free trade
the corporate sector and in the financial position agreement proposed by the USA with 11 other
of the sovereign governments. The corporate partners (Japan, Malaysia, Vietnam, Singapore,
loans are at a greater risk of defaults as further Brunei, Australia, New Zealand, Canada, Mexico,
loans are being raised by them to deal with the Peru, Chile) to facilitate lower tariffs and easy
cash shortages, further increased expenditure by movement of labor and investments. However,
the governments to deal with the pandemic the US withdrew from TPP in 2016 and the
has widened the fiscal deficits of the countries. remaining 11 countries revised TPP
to Comprehensive and Progressive Agreement
Q.47) Ans: b for Trans-Pacific Partnership or CPTPP.
Exp:
• Statement 1 is incorrect: Economic Survey 2020- Q.49) Ans: d
2021 highlights the potential of India to become Exp:
the pharmacy of the world. It highlights • Statement 1 is incorrect: The composition
that India’s pharmaceutical industry is third of SAARC and BIMSTEC differs greatly with each
largest in terms of volume while in terms of value other. SAARC or the South Asian Association for
it is 14th largest in the world. Further, India’s Regional Cooperation was established in 1985 at
exports in pharmaceuticals have doubled in the Kathmandu, Nepal. It comprises 8 countries: India,
past 10 years and India is at 11th position in terms Afghanistan, Pakistan, Nepal, Bhutan, Bangladesh,
of total share of world pharma exports as per Sri Lanka, and Maldives. Bay of Bengal Initiative
2019. for Multi Sectoral Technical and Economic
• Statement 2 is correct: USA is now the largest Cooperation or BIMSTEC was formed in 1997 with
e

export destination for Indian exports followed by Headquarters at Dhaka, Bangladesh. It has 7
in

China. A fall in the exports in 2020-21 have been members: India, Bangladesh, Bhutan, Nepal, Sri
nl

attributed to the fall in the exports of Petroleum, Lanka, Myanmar, and Thailand.
l.o

Oil and Lubricants exports. Drug formulations, • Statement 2 is incorrect: Mekong Ganga
biologicals, have shown consistent positive Cooperation was formed in 2000 and is an
ria

growth and is the second largest exported agreement with India and the five countries
e

commodity of the country. (Economic Survey through which river Mekong passes. These
at

2020-21). include Myanmar, Laos, Thailand, Vietnam, and


cm

Cambodia. River Mekong originates in China and


Q.48) Ans: b China is not a party to this agreement. The
ps

Exp: platform includes cooperation in the field of


• Statement 1 is incorrect: Comprehensive tourism, culture, education, transport, and
.u

Economic Partnership Agreement is a kind of Free communication.


w
w

PRAYAS TEST 24 26
w
Contact us :info@onlyias.com

OnlyIAS Nothing Else Visit :dpp.onlyias.in


Contact : +91-7007 931 912

Australia, and accounts for more than 25% of the


Q.50) Ans: b world’s agricultural exports.
Exp: • Statement 2 is incorrect: India is not a member of
• Statement 1 is incorrect: D 10 group was this grouping even though it has its maximum
proposed by the United Kingdom in 2020 as a population employed in agriculture and related
grouping of G7 countries along with 3 democratic activities.
countries to create a supply chain for 5G
technologies. Q.53) Ans: c
• Statement 2 is correct: The membership of D 10 Exp:
includes the G7 countries (USA, UK, France, • Statement 1 is correct: Data Free Flow with
Germany, Italy, Japan, and Canada) and the three Trust was proposed by Japan at the G 20 summit
democracies (India, Australia, and South Korea). in Osaka in 2019. It formed a part of the Osaka
G7 was originally in 1975 and with the addition of Declaration and aims to allow free flow of data
Russia in 1997, it became G8. However, post the between the countries to effectively harness the
annexation of Crimea by Russia in 2014, Russia fruits of the digital economy. India has abstained
was expelled, and it again reduced to G7. from this declaration as India is of the view that
• Statement 3 is correct: One of the objectives of data should be stored within the borders of the
the D 10 is to by-pass China in the supply of 5G countries.
technologies. It is done to ensure data privacy and • Statement 2 is incorrect: On the sidelines of
protection as Chinese companies would be the Buenos Aires Summit of the G 20, JAI trilateral
prevented from being a part of the supply chain. comprising Japan, USA, and India was formed to
• Statement 4 is incorrect: Australia, India, and promote economic growth among the three
South Korea are the three democratic countries. countries.
Japan is already a member of the G7 and hence a • Statement 3 is correct: Shanghai Cooperation
member of the group. Organization (SCO) was established in 2001 in
Beijing, China to promote growth and cooperation
Q.51) Ans: c among the central Asian countries. It comprises 6
Exp: founding countries: China, Russia, Tajikistan,
• Option C is correct: Asia Pacific Economic Kyrgyzstan, Kazakhstan, and Uzbekistan. India and
Cooperation (APEC) is a group of 21 members of Pakistan were added to the grouping in the 2017
the Pacific rim of ocean economies with its summit at Astana. Regional Anti-Terrorist
headquarters in Singapore. India is not a Structure is a component of this grouping. SCO
member of this grouping. Indian Ocean Rim aims to combat terrorism and establish peace in
Association (IORA) is a group of 22 countries to the region.
boost cooperation in the Indian Ocean region. It
was established in 1997 with its headquarters in Q.54) Ans: c
Mauritius with India as its member. Organization Exp:
of the Petroleum Exporting Countries (OECD) is a • Option C is correct: Maastricht Treaty and
group of oil exporting countries with its the Digital Single Market Strategy are related
e

headquarters in Austria. It was established in to European Union. European Union was created
in

1961. It includes 13 members. India is not a in 1993 with the signing of the Maastricht
nl

member of this. SAFTA or the South Asia Free Treaty to create a common political, economic
l.o

Trade Area is a trade agreement of the SAARC union that facilitates easy movement of goods,
nations. It includes all SAARC countries including services, and citizens. In 2002, it released its own
ria

India. currency, Euro. Post the exit of Britain from the


e

EU, it is now left with 27 members. The exit of


at

Q.52) Ans: b Britain was done post a referendum in the UK in


cm

Exp: 2016 that favored the exit and as per the


• Statement 1 is correct: It is a group of 19 provisions of the Lisbon Treaty. The creation
ps

agricultural exporting nations that aims to of Digital Single Market Strategy by 2020 is aimed
to promote various digital, E-commerce, and
.u

promote and liberalize the trade in agriculture.


The grouping was established in 1986 in Cairns, telecommunication services.
w
w

PRAYAS TEST 24 27
w
Contact us :info@onlyias.com

OnlyIAS Nothing Else Visit :dpp.onlyias.in


Contact : +91-7007 931 912

• Option A is the Correct answer.


Q.55) Ans: d o A major part of the government's fiscal deficit is
Exp: because it needs to pay interest on its previous
• Statement 1 is incorrect: Generalized System of accumulated debt. If we want to measure the
Preferences List is released by the USA and government's deficit excluding the interest
contains a list of countries the imports from which payment on the previous debt then it is called
would be subjected to reduced or zero import the primary deficit. The goal of measuring the
duties. It covers only selected commodities and primary deficit is to focus on present fiscal
not all the imports. India was removed from this imbalances.
list in 2019. o Primary Deficit = Fiscal Deficit - Net interest
• Statement 2 is incorrect: Global System of Trade liabilities
Preferences among Developing countries is an o So, primary deficit tells about the deficit in the
agreement under the United Nations Conference government's budget excluding the interest
on Trade and Development (UNCTAD). It was liabilities on the government's accumulated debt.
formed in 1988 among the developing and the
Least Developed Countries and aims to reduce Q.58) Ans: c
trade barriers among each other. India is a Exp:
member of this group. • Statement 1 is Incorrect: Revenue Deficit is the
difference between the government's revenue
Q.56) Ans: d expenditure and revenue receipts. Revenue
Exp: deficit neither creates assets nor reduces
o Public Debt in India includes only Internal and liabilities. Revenue Deficit implies that the
External Debt incurred by the Central government's current expenses are more than its
Government. Internal Debt includes liabilities current revenues and will have to use up the
incurred by resident units in the Indian economy savings of other sectors of the economy to finance
to other resident units. its consumption expenditure.
• Statement 1 is Correct:Market Stabilization • Statement 2 is Correct: Those expenses of the
Scheme (MSS) Bonds are Governed by a MoU government which either creates assets (physical
between the Government of India and the RBI. or financial) or reduces liabilities are called
MSS was created to assist the RBI in managing its capital expenditures. Capital expenditures include
sterilization operations. The Government borrows acquisition of land, building, machinery,
under this scheme from the RBI, while proceeds equipment, purchase of shares by the
from such borrowings are maintained in a government and loans and advances by the
separate cash account with the latter and is used central government to state and union territory
only for redemption of T-bills /dated securities governments, PSUs and other parties.
raised under this scheme. • Statement 3 is Correct: There are several grants
• Statement 2 is Correct: All deposits under small given by the Central Government to the States /
savings schemes are credited to the National UTs which come under revenue expenditure for
Small Savings Fund (NSSF). The balance in the the central government but some of these grants
e

NSSF (net of withdrawals) is invested in special create assets, which are owned by the State
in

Government securities and is counted as the government and not by the Central government.
nl

Internal Debt. Hence, for the Central Government it is basically


l.o

• Statement 3 is Correct: Securities issued to revenue expenditure but ultimately it is creating


International Financial Institutions such assets for the State government.
ria

as International Monetary Fund, International Hence, “effective revenue deficit" is calculated


e

Bank of Reconstruction and Development which excludes such grants which are used for
at

(IBRD), etc. for India’s contributions to these creation of assets.


cm

institutions are also included in the internal debt


of the country. Q.59) Ans: a
ps

Exp:
Q.57) Ans: a • Statement 1 is Incorrect: Fiscal deficit indicates
.u

Exp: the total borrowing of the government from all


w
w

PRAYAS TEST 24 28
w
Contact us :info@onlyias.com

OnlyIAS Nothing Else Visit :dpp.onlyias.in


Contact : +91-7007 931 912

sources i.e. domestic borrowing plus borrowing Q.62) Ans: d


from external sources. A large share of revenue Exp:
deficit in the fiscal deficit indicates that a large • Statement 1 is Incorrect: Fiscal deficit indicates
part of borrowing is being used to meet the total borrowing of the government from all
its consumption expenditure needs rather than sources i.e. domestic borrowing plus borrowing
investment. from external sources. However, a large part of
• Statement 2 is Correct: Revenue Deficit implies the government's fiscal deficit is the interest paid
that the government's current expenses are more on its previous accumulated debt.
than its current revenues and will have to use up o Fiscal Deficit = Total expenditure - total receipts
the savings of other sectors of the economy to except borrowing
finance its consumption expenditure. Since a • Statement 2 is Incorrect: Revenue Deficit is the
major part of the revenue expenditure (salary, difference between the government's revenue
pension, interest payments, subsidies etc.) is expenditure and revenue receipts.
committed expenditure, it cannot be reduced o Revenue Deficit = Revenue Expenditure -
after a certain extent and hence it cannot be Revenue Receipts
completely eliminated. o Revenue Deficit implies that the government's
current expenses are more than its current
Q.60) Ans: d revenues and will have to use up the savings of
Exp: other sectors of the economy to finance its
• Statement 1 is Incorrect: It was set up under the consumption expenditure. However, even in the
Central Goods and Services Tax (CGST) Act, case of revenue deficit, a major part goes for the
2017. The consumer welfare fund rules of 1992 payment of salaries, pensions, interest payments
have been subsumed under the CGST rules, 2017. on previously accumulated debt, subsidies which
It has been set up to promote and protect the is a committed expenditure.
welfare of the consumers. • Statement 3 is Correct: The government's deficit
• Statement 2 is Incorrect: The Fund has been set excluding the interest payment on the previously
up by the Department of Revenue (Ministry of accumulated debt is called the primary deficit. It
Finance) and is being operated by the was introduced with the goal of measuring the
Department of Consumer Affairs (Ministry of present fiscal imbalances.
Consumer Affairs, Food & Public Distribution). o Primary Deficit = Fiscal Deficit - Net interest
Q.61) Ans: a liabilities
Exp:
• Option A is the Incorrect statement. Q.63) Ans: d
o A buyback is a scheme by which a company Exp:
repurchases a certain amount of its outstanding • Option D is the Correct answer.
shares. Once taken back, these shares are o The G7 finance ministers agreed in principle upon
extinguished by the company. It can be done to global tax reforms, and this is being described as
improve the earnings per share for continuing historic by many. The agreement covers two
shareholders. It can also be done if promoters pillars: the first requiring MNCs to pay taxes in
e

want to hike their stake in the company, countries where they operate and not just where
in

sometimes to avoid any takeover threats. they have their headquarters; and the second
nl

o Companies resort to buyback mainly to escape pillar commits to a global minimum corporate tax
l.o

dividend distribution tax. To plug the differential of at least 15 percent on a country-by-country


tax treatment between buybacks and dividend basis.
ria

payouts, the government has proposed that 20 % o This has been done mainly because companies
e

tax will be levied on the difference between the preferred to book their profits in tax havens. For
at

issue price and the buyback price of the share. instance, U.S. companies booked half of all foreign
cm

The Finance Ministry has said that it will look into profits in just seven low-tax jurisdictions:
the applicability of 20% tax proposed in the 2019- Bermuda, the Cayman Islands, Ireland,
ps

20 Budget on the current share buybacks by listed Luxembourg, the Netherlands, Singapore and
companies. Switzerland. Though small countries levy a low
.u
w
w

PRAYAS TEST 24 29
w
Contact us :info@onlyias.com

OnlyIAS Nothing Else Visit :dpp.onlyias.in


Contact : +91-7007 931 912

rate, they may capture what is significant revenue various tax concessions and incentives provided
for them. under the Income tax Law.
o U.S. Treasury Secretary Janet Yellen has asserted o Minimum Alternate Tax is applied when the
that a global minimum would end a destructive taxable income calculated according to the I-T
“race to the bottom” in international taxation. Act provisions is found to be less than 15.5 per
According to the London-based Tax Justice cent (plus surcharge and cess as applicable) of the
Network advocacy group, governments lose $245 book profit under the Companies Act, 2013.
billion annually to tax havens. If that money were • Statement 4 is Incorrect: Stamp duty is a legal tax
instead available to governments, they could use payable in full and acts as evidence for any sale
it for, among other things, managing their heavy or purchase of a property. It is payable under
costs for pandemic relief. Section 3 of the Indian Stamp Act, 1899. Although
o It would apply to companies’ overseas it is applicable to the companies, it is not a
profits. Therefore, if countries agree on a global direct tax.
minimum, governments could still set whatever o The levy of stamp duty is a state subject and thus
local corporate tax rate they want. the rates of stamp duty vary from state to
state. Beginning from July, 2020, all shares and
Q.64) Ans: c mutual fund purchases will attract a stamp duty
Exp: of 0.005 per cent and any transfer of security will
o A direct tax is a tax levied directly on a taxpayer attract a stamp duty of 0.015 per cent.
who pays it to the Government and cannot pass it
on to someone else. Q.65) Ans: a
• Statement 1 is Correct: Corporate income tax Exp:
(CIT) is imposed on the profits of the • The Finance Ministry has permitted some states
corporations/ companies/ entities. All profit- an additional borrowing of ₹ 1.06 lakh Cr as these
making companies need to pay a flat rate of have implemented some institutional reforms.
corporate income tax. Last October, the central govt linked
• To promote manufacturing, for new the additional borrowing of 1% of GSDP to four
manufacturing firms (& electricity generation) set critical reforms:
up after 1st October 2019 and commencing its o One Nation One Ration Card
operations before 31st March 2023, the Standard o Ease of doing business reforms
Corporate Income Tax is 15% (after Cess and o Power sector reforms
Surcharge it will be 17.16%). o Urban and local body reforms
• Statement 2 is Incorrect: When a company • States implementing three of these are given
announces dividends, it has to pay tax (DDT) on additional grants from the centre for capex
the dividend which is to be distributed to the purposes. This was done under the AtmaNirbhar
owners and the owners also pay tax (as per their Bharat, the central govt announced support
income tax slab) on the dividend received. measures allowing them to raise the borrowing
• However, from Assessment Year 2021-22, the limit from 3% to 5%, which would translate to
domestic company isn't required to pay dividend additional borrowing of ₹ 4.28 lakh Cr. However,
e

distribution tax on any amount declared, the Centre had attached conditions for the
in

distributed or paid by such company by way of increased borrowing space, they are:
nl

dividend. Dividends received from domestic o Permitting only 0.5 per cent of GSDP as an
l.o

companies are taxable in the hands of unconditional increase.


shareholders. o The next 1 per cent was to be in four tranches of
ria

• Statement 3 is Correct: Companies try to 0.25 per cent, with each tranche linked to
e

minimise giving taxes by taking advantage of expenditure on ‘One Nation One Ration’, urban
at

depreciation, deductions, exemptions, etc from local body revenues, power distribution, ease of
cm

the government. The objective of introduction of doing business reforms.


MAT was to bring into the tax net "zero tax o The last 0.5 per cent was to be allowed if at least
ps

companies" which in spite of having earned three out of four milestones are reached.
substantial book profits and having paid o States will also have to agree to take over the
.u

handsome dividends, do not pay any tax due to future losses of state power distribution
w
w

PRAYAS TEST 24 30
w
Contact us :info@onlyias.com

OnlyIAS Nothing Else Visit :dpp.onlyias.in


Contact : +91-7007 931 912

companies in a graded manner to be eligible for • Statement 2 is Incorrect: In case of composition


additional borrowing space of 0.50% of GSDP for scheme, the businesses can’t claim input tax
four years. credit and it is an optional scheme as they will
o For FY22, the net borrowing ceiling for states has always have the freedom to pay standard GST
been fixed at 4 percent of the projected rate. The eligible person opting to pay tax under
GSDP (about Rs 8.46 lakh crore), based on this scheme can pay tax at 1% flat rate, of his
recommendations of the Fifteenth Finance turnover, instead of paying tax at normal GST
Commission. The states that have implemented rate. Similarly, small service providers with
all four reforms include Andhra Pradesh, Goa, turnover of Rs. 50 lakhs can opt for composition
Kerala, Madhya Pradesh, Punjab, Rajasthan, scheme and pay GST at 6%.
Telangana and Tripura.
Q.68) Ans: b
Q.66) Ans: c Exp:
Exp: • Statement 1 is Incorrect: Govt. of India promised
o Relevance: Beginning from July, 2020, all shares States that if after implementation of GST, the
and mutual fund purchases will attract a stamp States Indirect Revenue growth will be less than
duty of 0.005 per cent and any transfer of 14% annually from 2015-16 (base year) onwards
security will attract a stamp duty of 0.015 per then Govt. of India (through GST council) will
cent. impose Cess(which is a tax on tax) on luxury and
• Statement 1 is Correct: As per the recent demerit goods and that money it will give to
government notification, all shares and mutual States as compensation for the next five years
fund purchases will attract a stamp duty of 0.005 (2017-22). For this purpose Goods and Services
per cent. Tax (Compensation to States) Act 2017 was
• Statement 2 is Correct: Stamp duty is the tax enacted.
levied by the State government( as it is a State • Statement 2 is Correct: Currently, only states can
Subject) when a property is transferred from a be compensated for the revenue loss that may
seller to a buyer. Only when the receipt or occur due to the introduction of GST but there
acknowledgement of payment of stamp duty is is no such provision for the Union government.
received, will one be considered a legal owner of
that property. Q.69) Ans: b
• Stamp duty is payable irrespective of the type of Exp:
property purchased — under-construction, new • Statement 1 is Correct: Article 265 of the
completed or re-sale. Constitution provides that no tax shall be levied or
• Statement 3 is Correct: From July, 2020, stamp collected except by authority of law. Government
duty will also be applicable to any transfer of takes the approval of the parliament for the
security, shares or debentures. It will attract a taxes/receipts through the Finance Bill and the
stamp duty of 0.015 per cent. approval for the expenditures through the
Appropriation Bill.
Q.67) Ans: c • Statement 2 is Incorrect: The pro-cyclical fiscal
e

Exp: policy goes in line with the current mood of the


in

• Statement 1 is Incorrect: Composition levy is not business cycle i.e. amplifying them. For example,
nl

a direct tax. The objective of the composition during a recession, the government reduces
l.o

scheme is to bring simplicity and to reduce the spending and increases taxes, leading to further
compliance cost for the small taxpayers. Small slowdown in the economy. This kind of fiscal
ria

taxpayers with an aggregate turnover in a policy is dangerous and brings instability in the
e

preceding financial year up to 1.5 Cr. shall be economy.


at

eligible for composition levy. • When the government reduces taxes during a
cm

o Suppliers opting for composition levy need not recession it follows a counter-cyclical fiscal policy.
worry about the classification of their goods or • Statement 3 is Correct: GST is also
ps

services or both, the rate of GST applicable on the called consumption based and destination-based
same, etc. They are not required to raise any tax
.u

tax as all the GST is passed on to the consuming


invoice, but simply need to issue a Bill of Supply.
w
w

PRAYAS TEST 24 31
w
Contact us :info@onlyias.com

OnlyIAS Nothing Else Visit :dpp.onlyias.in


Contact : +91-7007 931 912

State rather than the state which produces the underpinning assumptions and risks. It is not
goods. presented along with the budget.
• Statement 4 is Correct: One of the ways of • Statement 3 is Correct: The Annual Financial
classification of taxes is- Specific tax and Ad- Statement is a document presented to Parliament
valorem tax. Specific tax is the tax which is fixed every financial year as part of the Budget
as per each unit of good or service rather than process, as required under Article 112 of the
based on its value. Ad-valorem or the value Constitution of India. The documents consist of
added tax is levied as a percentage of value of receipts and expenditures of the government in
the item it is imposed on, and not on the item's the current year, previous year and the Budget
quantity, size, weight or other such factor. year in three separate parts — Consolidated Fund
o Hence, the collection of a value added tax like of India, Contingency Fund of India, and Public
GST is mainly based on the growth in Account of India.
the Nominal GDP. Whereas, that of a specific tax • Statement 4 is Correct: Article 113 of the
is based on the Real GDP because it is charged as Constitution mandates that the estimates of
per each unit of a particular good. expenditure from the Consolidated Fund of India
included in the Annual Financial Statement and
Q.70) Ans: b required to be voted by the Lok Sabha be
Exp: submitted in the form of Demands for
• Statement 1 is Incorrect: According to the Fiscal Grants. The Demands for Grants are presented to
Responsibility and Budget Management Act, the Lok Sabha along with the Annual Financial
2003, the Finance Minister shall review, on half- Statement.
yearly basis, the trends in receipts and • Statement 5 is Incorrect: When
expenditure in relation to the budget and place the amount authorized to be expended for the
before both Houses of Parliament the outcome of current financial year is found to
such reviews. be insufficient for the purpose of that year or
• Statement 2 is Correct: Budget 2021-22 when a need has arisen
announced that the FRBM Act will be amended during the current financial year for suppleme
and the government plans to continue on the ntary or additional expenditure upon some
path of fiscal consolidation, achieving a fiscal 'new service' not contemplated in the budget
deficit level below 4.5% of GDP by 2025-2026 for that year then the President causes to be
with a fairly steady decline over the laid the "Supplementary Demand for Grants".
period. Moreover, in light of the Covid-19 o The "Supplementary Demands for Grants" are
pandemic, the fiscal deficit in BE 2021-2022 is presented to and passed by the House before the
estimated to be 6.8% of GDP. The fiscal deficit in end of the financial year and not along the Union
RE 2020-21 is pegged at 9.5% of GDP. budget.

Q.71) Ans: d Q.72) Ans: b


Exp: Exp:
• Statement 1 is Correct: The Fiscal Policy Strategy o To understand Goods and Services tax,
e

Statement, presented to Parliament understanding the exemptions under it is equally


in

under Section 3(4) of the Fiscal Responsibility important. Following goods and services are
nl

and Budget Management (FRBM) Act, exempt from the application of the GST:
l.o

2003, outlines the strategic priorities of the • Edible vegetables, roots and tubers
government in the fiscal area for the ensuing • Cereals
ria

financial year relating to taxation, expenditure, • Fish (not frozen or processed)


e

lending and investments, administered pricing, • Fresh fruits and vegetables (other than frozen or
at

borrowings and guarantees. It is to be presented processed)


cm

along with the budget. • Meat (other than frozen or put up in unit
• Statement 2 is Incorrect: The medium-term containers)
ps

expenditure framework (MTEF) statement sets a • Cane jaggery (Gur)


three-year rolling target for expenditure • Tender coconut water
.u

indicators, along with specifications of • Silkworm laying cocoon


w
w

PRAYAS TEST 24 32
w
Contact us :info@onlyias.com

OnlyIAS Nothing Else Visit :dpp.onlyias.in


Contact : +91-7007 931 912

• Raw silk Bharat Mission (SBM), Pradhan Mantri Awas


• Silk waste Yojana (PMAY), etc were financed out of extra
• Wood, not carded or combed budget borrowings.
• Cotton used in Gandhi topi • Main entities that have borrowed are FCI,
• Cotton used in khadi yarn National Highway Authority of India, National
• Coconut, coir fibre Bank for Agriculture and Rural Development
• Jute fibre (raw and processed but not spurn) (NABARD), which has borrowed for both rural
• Pooja Samagri development and irrigation projects.
• Live animals except horses • The Fifteenth Finance Commission in its initial
• All goods of seed quality report advised both the centre and the states to
• Unroasted cotton beans eliminate extra budget borrowings.
• Unprocessed green tea leaves • The Commission noted that there is an increasing
• Fresh ginger and fresh turmeric (not processed) tendency of the Union and State Governments to
• Human blood and its components borrow outside the Consolidated Fund, leading to
• All types of contraceptives accumulation of extra-budgetary liabilities.
• Organic manure (without the brand name) • The Commission recommended that in the
• Kumkum, bindi, sindur, alta interest of transparency, both the Union and the
• Firewood or fuelwood States have to make full disclosure of extra-
• Wood charcoal budgetary borrowings. Outstanding extra-
• Betel leaves budgetary liabilities need to be clearly identified
• Judicial and non judicial stamp papers, court fee and eliminated in a time-bound manner as per
stamps when sold by the government treasuries the amended FRBM Act of 2018.
or authorized vendors
• Postal items sold by the government Q.74) Ans: b
• Rupee Notes when sold to the RBI and cheques Exp:
• Printed books, braille books, newspapers, maps o External Debt of India refers to all the external
• Earthen pots and clay lamps debt taken either by central govt or state govt or
• Bangles (except those made from precious any Indian company. As on 31st March 2020,
materials) external debt of India was $ 558.5 Billion (20.6%
• Manually operated or animal driven agricultural of GDP).
implements • Statement 1 is Incorrect: Short term debt (up to
• Hand tools, spades, shovels one year)
• Handloom is 106.9billionandthelongtermdebtis106.9billiona
• Spacecraft ndthelongtermdebtis 451.7 billion. Hence, it is
• Hearing aids less than the long term debt. This again includes
• Electricity both sovereign and non-sovereign debt but there
is no separate data available. It includes FII
Q.73) Ans: b investment in Treasury bills and Indian corporate
Exp: bonds etc.
e

• Option B is the Correct answer. • Statement 1 is Correct: Commercial borrowings


in

• Off-budget borrowings are loans that are taken remained the largest component of external
nl

not by the Centre directly, but by another public debt, with a share of 37.4 per cent, followed by
l.o

institution which borrows on the directions of the non-resident deposits (24.9 percent) and short-
central government. The repayment of the entire term trade credit (17.1 per cent).
ria

principal and interest is done from the


e

Government budget. Q.75) Ans: c


at

• Extra budget borrowing is excluded from the fiscal Exp:


cm

deficit calculations, but at the same time, are Option C is the Correct answer.
added to the total debt of the government. • A fiscal council is a permanent agency with a
ps

• Schemes like the Pradhan Mantri Krishi mandate to independently assess the
Sinchayee Yojana (PMKSY), Deen Dayal
.u

government’s fiscal plans and projections against


Upadhayay Gram Jyoti Yojana (DDUGJY), Swachh
w
w

PRAYAS TEST 24 33
w
Contact us :info@onlyias.com

OnlyIAS Nothing Else Visit :dpp.onlyias.in


Contact : +91-7007 931 912

parameters of macroeconomic sustainability, earliest. Such debt is not sustainable (for


and put out its findings in the public domain. instance, the economic crisis in Greece).
• Historically, budgets in India have consistently • However, since the interest rate on debt paid by
overestimated revenue growth and the Indian Government has been less than India’s
underestimated expenditure growth. The finance growth rate, in the Indian context, growth leads
ministry’s overall record in forecasting to debt sustainability but not necessarily vice-
projections has been consistently poor under versa. Debt sustainability depends on the
successive finance ministers. difference between the interest rate and the
• Countries with independent fiscal councils tend to growth rate in the economy.
produce relatively more accurate budget
forecasts and stick better to fiscal rules. Q.77) Ans: d
• In 2017, the N.K. Singh's committee on the review Exp:
of fiscal rules set up by the finance ministry • Relevance: US Trade Representative (USTR) has
suggested the creation of an independent released the findings of the Section 301
fiscal council that would provide forecasts and report which said that India’s 2% equalisation
advise the government on whether conditions levy is unreasonable or discriminatory potentially
exist for deviation from the mandated fiscal rules. attracting withdrawal of US trade concessions or
• In 2018, the D.K. Srivastava committee on fiscal duties on Indian exports.
statistics established by the National Statistical • Statement 1 is Incorrect: Equalization Levy is not
Commission (NSC) also suggested the under the Income Tax law as tax on income,
establishment of a fiscal council that could rather as an independent levy introduced
coordinate with all levels of government to through Finance Act 2016. Under the Finance Bill
provide harmonized fiscal statistics across 2020-2, a 2% digital service tax (DST) was also
governmental levels and provide an annual imposed on non-resident e-commerce operators
assessment of overall public sector borrowing in India. Companies with a turnover of over Rs. 2
requirements. crore, will pay this levy on the consideration
• These recommendations follow similar received for online sales of goods and services.
recommendations from the 13th and 14th finance • Equalisation levy was one of the methods
commissions, which also advocated the suggested by the 2015 OECD/G20 Report on
establishment of independent fiscal agencies. Action 1 of BEPS Project which was aimed at
tackling the taxation challenges arising out of
Q.76) Ans: a digitization of the economy.
Exp: • Statement 2 is Incorrect: Equalisation levy is
o Debt sustainability depends on the “interest rate a Direct Tax and is levied on "REVENUE" and not
growth rate differential” (IRGD) i.e. the difference "Profit". This is because for India it will be very
between the interest rate and the growth rate in difficult to check the profit of a multinational
the economy. company (for instance, Google) as it has a lot of
• Statement 1 is Correct: Generally when the operations in the US and other countries.
economy grows at a higher rate, the • Statement 3 is Incorrect: The Equalization levy
e

difference between the interest rate on debt and levied at 2% is applicable on non-resident e-
in

growth rate will be negative. As the IRGD is commerce operators, not having a permanent
nl

expected to be negative in the foreseeable future establishment or any tangible presence in


l.o

for India, a fiscal policy that provides an impetus India. Companies having a tangible presence in
to growth will lead to lower, not higher, debt to India have to pay income tax on their respective
ria

GDP ratio. This is the reason Eco Survey has earnings.


e

demonstrated counter-cyclical fiscal policy to o This was done mainly because in absence of the
at

enable growth during economic downturns like EL, non-resident e-commerce operators (not
cm

Covid-19. having any Permanent Establishment in India but


• Statement 2 is Incorrect: If the government has to significant economic presence) are not required to
ps

pay more interest on the debt than the growth pay taxes in respect of the consideration received
.u

rate, then it would be damaging to the economy in the e-commerce supply or services made in the
and it is advised to stop such borrowings at the Indian market.
w
w

PRAYAS TEST 24 34
w
Contact us :info@onlyias.com

OnlyIAS Nothing Else Visit :dpp.onlyias.in


Contact : +91-7007 931 912

Laundering act, 2002, may file an application


Q.78) Ans: d before a Special Court (designated under the
Exp: PMLA) to declare a person an FEO. The Special
Option D is the Correct answer. Court will issue a notice to the individual: (i)
• The Fiscal Responsibility and Budget Management requiring him to appear at a specified place on a
(FRBM) Act was passed in 2003 and it sets the date which is at least six weeks after the issue of
fiscal rules to be followed by the government to notice, and (ii) stating that a failure to appear will
ensure fiscal consolidation. result in him being declared an FEO. Notice may
• In 2018, the Act was amended after a review of also be served at the individual’s email address
the Act made by a committee under Dr NK recorded in the PAN or Aadhaar databases.
Singh. The amendment added a new road map • Statement 3 is correct: The Special Court may
and timeline to reduce the fiscal deficit to 3% of confiscate properties of an FEO which are
GDP by 31st March 2021. It also inserted the proceeds of crime, benami properties, or any
escape clause that allows the government to go other properties. These properties may be in
above the fiscal deficit target set for the year India or abroad. The Special Court may exempt
during some extraordinary circumstances. certain properties from confiscation where any
• The subsection 4 (2) of the Act says about various other person has a legitimate interest in
grounds on which the FRBM’s fiscal deficit target them. Upon confiscation, all rights and titles in
may be exempted during a year. Exceeding annual the property will vest in the central government,
fiscal deficit target due to the following ground or free from encumbrances (claims or rights in the
grounds may be permitted: property). The central government may dispose
o National security or act of war of these properties after 90 days.
o National calamity
o Collapse of agriculture severely affecting farm Q.80) Ans: c
output and incomes Exp:
o Structural reforms in the economy with o An Act to ensure financial stability and promote
unanticipated fiscal implications competitiveness in Indian financial markets by
o Decline in real output growth of a quarter by at providing enforceability of bilateral netting of
least three per cent points below its average of qualified financial contracts and for matters
the previous four quarters So, the escape clause connected therewith or incidental thereto.
of the FRBM allows the government to borrow • Statement 1 is correct: Bilateral netting: Netting
more than the statutorily allowed limit during refers to offsetting of all claims arising from
extraordinary circumstances. dealings between two parties, to determine a net
amount payable or receivable from one party to
Q.79) Ans: c other. In the event of the default or insolvency of
Exp: one of the parties, a bank’s obligation would be
• Statement 1 is incorrect: Fugitive economic the net sum of all positive and negative fair values
offender (FEO): An FEO is a person against whom of contracts included in the bilateral netting
an arrest warrant has been issued for committing arrangement.
e

any offence listed in the Act, and the value of the • Statement 2 is correct: Qualified financial
in

offence is at least Rs 100 crore. Further, the contracts (QFC): QFC means any bilateral contract
nl

person has left the country and refuses to return, notified as a QFC by the relevant authority. The
l.o

in order to avoid facing prosecution. The Act lists authority can be Reserve Bank of India (RBI),
55 economic offences in the Schedule, which Securities and Exchange Board of India (SEBI),
ria

include: (i) counterfeiting government stamps or Insurance Regulatory and Development Authority
e

currency, (ii) dishonouring cheques, (iii) benami of India (IRDAI), Pension Fund Regulatory and
at

transactions, (iv) transactions defrauding Development Authority (PFRDA) or International


cm

creditors, (v) tax evasion, and (vi) money- Financial Services Centres Authority (IFSCA). The
laundering. The central government may amend Central government may, by notification, exclude
ps

the Schedule through a notification. contracts between certain parties or containing


• Statement 2 is incorrect: The authorities certain terms from being designated as QFCs.
.u

appointed under the Prevention of Money


w
w

PRAYAS TEST 24 35
w
Contact us :info@onlyias.com

OnlyIAS Nothing Else Visit :dpp.onlyias.in


Contact : +91-7007 931 912

Q.81) Ans: a village industries, handicrafts and other rural


Exp: crafts.
• Option a is correct: Demutualised means that the • Support all other allied economic activities in rural
stock brokers owned organization have been areas, promote integrated and sustainable
made a public owned organization. The shares in rural development and secure prosperity of rural
the hand of the brokers were transferred to the areas.
public and this process is known as • Option a is correctly mentioned: The importance
demutualisation. For example, Bombay Stock of institutional credit in boosting the rural
Exchange established in 1887 was known as ‘ The economy has been clear to the Government of
Native Share and Stock Brokers Association’ and India right from its early stages of planning.
was owned by stock brokers before it was Therefore, the Reserve Bank of India (RBI) at the
demutualised. insistence of the Government of India, constituted
a Committee to Review the Arrangements For
Q.82) Ans: d Institutional Credit for Agriculture and Rural
Exp: Development (CRAFICARD) to look into these very
• Securities and Exchange Board of India is a quasi- critical aspects. The Committee was formed on 30
legislative, quasi-judicial and quasi-executive body March 1979, under the Chairmanship of Shri B.
and can draft regulations, conduct inquiries, pass Sivaraman, former member of Planning
rulings and impose penalties. Commission, Government of India.
• Securities and Exchange Board of India is an • Option b is correctly mentioned: Promotes
autonomous organization that works under sustainable and equitable agriculture and rural
overall administrative supervision of the Union development through participative financial and
Finance Ministry and is accountable to the non-financial interventions, innovations,
Parliament. technology and institutional development for
• SAT is a statutory body established under the securing prosperity.
provisions of the Securities and Exchange Board of • Option d is correctly mentioned: National Bank
India Act, 1992, it was constituted to protect the for Agriculture and Rural Development (NABARD)
interest of entities that feel aggrieved by SEBI’s has released a study named, ‘Achieving
decision can also hear and dispose of appeals Nutritional Security in India: Vision 2030’, in
against orders passed by the Insurance Regulatory December 2020. The report assesses the trends
Development Authority of India (IRDAI) under the for nutritional security and identifies determining
Insurance Act, 1938, the General Insurance factors that have a significant effect on reducing
Business (Nationalization) Act, 1972 and the malnutrition levels in India.
Insurance Regulatory and Development Authority
Act, 1999. Hence option a, b and c are correctly Q.84) Ans: c
mentioned. Exp:
• The chairman of SEBI is nominated by the Union • Option c is correct : The Government of India
Government of India.The other members encourages farmers in taking up projects in select
include:Two members, i.e., Officers from the areas by subsidizing a portion of the total project
e

Union Finance Ministry, One member from the cost. All these projects aim at enhancing capital
in

Reserve Bank of India and the remaining five investment, sustained income flow and
nl

members are nominated by the Union employment areas of national importance.


l.o

Government of India. Three of the five members NABARD has been a proud channel partner of the
should be full-time members. Hence, option d is Government in some of these schemes shown in
ria

incorrectly mentioned. this section. Subsidy as and when received from


e

the concerned Ministry is passed onto the


at

Q.83) Ans: c financing banks.


cm

Exp: o Dairy Entrepreneurship Development Scheme


• Option c is incorrectly mentioned : Facilitate o Commercial production units of organic inputs
ps

credit flow for promotion and development of o Agriclinic and Agribusiness Centres Scheme
o National Livestock Mission
.u

agriculture, small-scale industries, cottage and


o GSS – Ensuring End Use of Subsidy Released
w
w

PRAYAS TEST 24 36
w
Contact us :info@onlyias.com

OnlyIAS Nothing Else Visit :dpp.onlyias.in


Contact : +91-7007 931 912

o Interest subvention Scheme • Option b is correct: Fruits and vegetables, cereals,


o New Agricultural Marketing Infrastructure. meat and fish (not frozen), potatoes and other
o The Department of Agriculture has been edible tubers and roots, tender coconut, tea
implementing the Centrally Sponsored Scheme, leaves, jaggery, coffee beans, ginger, turmeric,
“Paramparagat Krishi Vikas Yojana” (PKVY) since milk, curd are exempted goods under the GST
2015-2016. Hence option a, b and d are incorrect. exemption list.
To promote natural resource based integrated • Options a, c and d are incorrect: Processed foods
and climate resilient sustainable farming systems with longer shelf life will attract a higher rate of
that ensure maintenance and increase of soil GST than ready to eat food products. For example
fertility, natural resource conservation, on-farm plain roti or paratha will be taxed with a lower
nutrient recycling and minimize dependence of rate of GST than packed paratha and Milk in a
farmers on external inputs are the main objectives tetra pack. Similarly fresh food items are
of the scheme exempted from GST but processed and canned
food items are not.
Q.85) Ans: d
Exp: Q.87) Ans: c
• Statement 1 is correct: GST Bill was first Exp:
introduced in 2014 as The Constitution (122nd • Statement 1 is correct: The RBI shall recommend
Amendment) Bill.This got an approval in 2016 and the denominational values of the notes as well as
was renumbered in the statute by Rajya Sabha as the discontinuance of issue of notes (Section 24 of
The Constitution (101st Amendment) Act, 2016. the Reserve Bank of India Act, 1934). Further, it is
GST has replaced various other indirect taxes that on the recommendation of the RBI that the
were imposed on goods and services before. Central government may declare that any series
• Statement 2 is correct: Article 279A of the of bank notes shall cease to be legal tender
Constitution of India provides for formation of a (Section 26)
GST Council to be formed by The President to • Statement 2 is correct: Custodian of Foreign
administer & govern GST. The Chairman is the Reserves:-For the purpose of keeping the foreign
Union Finance Minister of India with ministers exchange rates stable, the Reserve Bank buys and
nominated by the state governments as its sells foreign currencies and also protects the
members. The decisions of the council are taken country's foreign exchange funds. RBI sells the
by 3/4th majority with the centre having 1/3rd foreign currency in the foreign exchange market
voting power and the states having 2/3rd. when its supply decreases in the economy and
• Statement 3 is correct: As per GST Notification vice-versa. Currently, India has a Foreign Exchange
05/2021, e-Invoicing is compulsory to generate for Reserve of around US$ 598 billion.
entities whose aggregate turnover is more than • Statement 3 is incorrect: To maintain price
50 crore. Earlier this limit was 100 crore. But now stability, inflation needs to be controlled. The
the government has reduced the threshold value Government of India sets an inflation target for
of e-invoices because they are trying to bring the every five years. RBI has an important role in the
medium to small scale companies under the consultation process regarding inflation targeting.
e

mandate of invoices. CBIC has notified vide The current inflation-targeting framework in India
in

Central Tax Notification no.23 dated 1st June 2021 is flexible in nature. The amended RBI Act
nl

that the e-invoicing system shall not apply to a provides for the inflation target to be set by the
l.o

government department and local authority. Government of India, in consultation with the
Reserve Bank, once every five years.
ria

Q.86) Ans: b • Statement 4 is correct: The Reserve Bank of India


e

Exp: (RBI) is vested with the responsibility of


at

o The Goods and Services Tax (GST) is a value-added conducting monetary policy. This responsibility is
cm

tax levied on most goods and services sold for explicitly mandated under the Reserve Bank of
domestic consumption. The GST is paid by India Act, 1934.The primary objective of monetary
ps

consumers, but it is remitted to the government policy is to maintain price stability while keeping
by the businesses selling the goods and services.
.u

in mind the objective of growth.


w
w

PRAYAS TEST 24 37
w
Contact us :info@onlyias.com

OnlyIAS Nothing Else Visit :dpp.onlyias.in


Contact : +91-7007 931 912

Q.88) Ans: b person company. It may have only one director


Exp: and one shareholder. The Companies Act
• Statement 1 is correct: The RBI undertakes the 1956 requires minimum two shareholders and
responsibility of controlling credit created by two directors in case of a private company.
commercial banks. RBI uses two methods to • Statement 3 is incorrect: The maximum number
control the extra flow of money in the economy. of persons/partners in any
These methods are quantitative and qualitative association/partnership may be upto such number
techniques to control and regulate the credit flow as may be prescribed but not exceeding one
in the country. When RBI observes that the hundred. This restriction will not apply to an
economy has sufficient money supply and it may association or partnership, constituted by
cause an inflationary situation in the country then professionals like lawyer, chartered accountants,
it squeezes the money supply through its company secretaries, etc. who are governed by
tight monetary policy and vice versa. their special laws. Under the Companies Act
• Statement 2 is correct: Section 45 of Banking 1956, there was a limit of maximum
Regulation Act 1949 empowers the Reserve Bank 20 persons/partners and there was no exemption
to make a scheme of amalgamation of a bank with granted to the professionals.
another bank if it is in the depositors' interest or • Statement 4 is correct: Under the Companies Act
in the interest of the overall banking system. 1956, a director had fiduciary (legal or ethical
Many private sector banks have been merged relationship of trust) duties towards a company.
with other private sector banks or the PSBs under However, the Companies Act 2013 has defined
this mechanism. The power of winding up of the duties of a director.
banks lies in the hand of the Reserve bank of • The other amendmendments include:
India. At the same time RBI can also apply for the • The Companies Act 2013 stipulates appointment
winding up of the bank if it thinks feet for the of at least one woman Director on the Board (for
bank. The Banking (Regulation) Act, 1949. The certain class of companies).
provision from Section 38 to 44 deals with the • The Companies Act 2013 stipulates certain class of
winding up procedure for banking companies. Companies to spend a certain amount of money
• Statement 3 is incorrect: Banks Boards bureau is every year on activities/initiatives reflecting
entrusted with search and select apposite Corporate Social Responsibility.
personages for Board of Public Sector Banks, • The Companies Act 2013 proposes a fast track and
Public Sector Financial Institutions and Public simplified procedure for mergers and
Sector Insurance Companies and recommend amalgamations of certain class of companies such
measures to improve Corporate Governance in as holding and subsidiary, and small companies
these Institutions. after obtaining approval of the Indian government

Q.89) Ans: a Q.90) Ans: c


Exp: Exp:
• The Indian Companies Act 2013 replaced the • State Rooftop Solar Attractiveness Index: State
Indian Companies Act, 1956. The Companies Act Rooftop Solar Attractiveness Index – SARAL is an
e

2013 makes comprehensive provisions to govern initiative of the Ministry of New and Renewable
in

all listed and unlisted companies in the country. Energy (MNRE) and its partners to introduce a
nl

The Companies Act 2013 implemented many new measuring scale or an index to evaluate and rank
l.o

sections and repealed the relevant corresponding all states according to their performance, growth,
sections of the Companies Act 1956. This is a
ria

level of maturity, policy framework, and


landmark legislation with far-reaching implementation environment in the rooftop solar
e

consequences on all companies incorporated in sector.


at

India. • Health Index of India: The health index has been


cm

• Statement 1 is correct: The Companies Act 2013 developed by NITI Aayog, with technical
increased the number of maximum shareholders assistance from the World Bank, in consultation
ps

in a private company from 50 to 200. with the Ministry of Health and Family Welfare.
• Statement 2 is correct: The Companies Act 2013 • AMFRI Index: NITI Aayog has released the
.u

provides a new form of private company, i.e., one Agricultural Marketing & Farm Friendly Reforms
w
w

PRAYAS TEST 24 38
w
Contact us :info@onlyias.com

OnlyIAS Nothing Else Visit :dpp.onlyias.in


Contact : +91-7007 931 912

Index (AMFFRI) 2019. The first-ever index was placed by the e-commerce entity. This would help
released in October. AMFFRI is an index which create a database of genuine e-commerce entities
compares the status of reforms in the agriculture and ensure that the consumers are able to verify
sector across all the states and UTs in the country. the genuineness of an e-commerce entity before
• Sustainable Tourism in Himalayan transacting through their platform.
Region: Sustainable Tourism in the Indian
Himalayan Region released by the NITI Aayog in Q.92) Ans: a
August this year, except for Sikkim and Himachal Exp:
Pradesh, the environmental index of the rest of • Option a is correct: The New Arrangement to
Himalayan states is very low. The report notes Borrow is the fund mobilization arrangement of
that the tourism and hospitality sector is a highly the IMF to mobilise additional funds through
lucrative one as far as the Indian Himalayan borrowing from member countries. Under NAB,
Region (IHR) is concerned. The sector directly member countries and institutions can stand
contributes about US$ 71.5 billion to the nation’s ready to lend to the Fund. The NAB is structured
GDP. as a set of credit arrangements between the IMF
• Digital Payments Trends, Issues and and 38-member countries (can be changed) and
Opportunities: A report on Digital Payments was institutions.
prepared by NITI Aayog and released in July 2017 • The NAB was proposed for the first time at the
for the first time. Primary objective of the report 1995 G-7 Halifax Summit following the Mexican
was to provide relevant data on the growth of financial crisis. In January 1997, the IMF’s
digital payments so that policy makers can Executive Board adopted a decision establishing
monitor the progress of digital payments in the the NAB, which became effective in November
country. 1998.
• The NAB was revived in the context of the global
Q.91) Ans: c financial crisis in 2009; to meet the lending
Exp: requirements of the Eurozone crisis.
• Statement 1 is incorrect: Conventional flash sales
by third party sellers are not banned on e- Q.93) Ans: a
commerce platforms. But, certain e-commerce Exp:
entities are engaging in limiting consumer choice • Option a is incorrect. Asian Development Bank
by indulging in “back to back” or “flash” sales (ADB) was established in the year 1966, with head
wherein one seller selling on platform does not office at Manila (Philippines). It has 67 members
carry any inventory or order fulfilment capability from the Asia Pacific region. This bank was
but merely places a “flash or back to back” order modeled on the lines of the world bank. Japan
with another seller controlled by platform. This holds the largest share in ADB with 15.677%,
prevents a level playing field and ultimately limits followed by U.S.A (15.567%), China (6.473%), and
customer choice and increases prices. India (5.812%). China is the largest shareholder
• Statement 2 is correct: To ensure that the followed by India in Asian Infrastructure and
domestic manufacturers and suppliers get a fair Invest Bank and not in ADB.
e

and equal treatment on the e-commerce platform • ADB is committed to achieving a prosperous,
in

it has been provided that where an e-commerce inclusive, resilient, and sustainable Asia and the
nl

entity offers imported goods or services, it shall Pacific, while sustaining its efforts to eradicate
l.o

incorporate a filter mechanism to identify goods extreme poverty. Established in 1966, it is owned
based on country of origin and suggest by 68 members—49 from the region.
ria

alternatives to ensure fair opportunity to • ADB was conceived in the early 1960s as a
e

domestic goods. financial institution that would be Asian in


at

• Statement 3 is correct: Framework for character and foster economic growth and
cm

registration of every e-commerce entity with the cooperation in one of the poorest regions in the
DPIIT ( Department for Promotion of Industry and world.A resolution passed at the first Ministerial
ps

Internal Trade) for allotment of a registration Conference on Asian Economic Cooperation held
number which shall be displayed prominently on
.u

by the United Nations Economic Commission for


the website as well as invoice of every order
w
w

PRAYAS TEST 24 39
w
Contact us :info@onlyias.com

OnlyIAS Nothing Else Visit :dpp.onlyias.in


Contact : +91-7007 931 912

Asia and the Far East in 1963 set that vision on the Directors shall be from Brazil. The first President
way to becoming reality. of the Bank shall be from India. The headquarters
• ADB's highest policy-making body is the Board of of the Bank shall be located in Shanghai. The New
Governors, which comprises one representative Development Bank Africa Regional Center shall be
from each member nation – 48 from the Asia- established in South Africa concurrently with the
Pacific and 19 from outside the region. Hence headquarters.” (Fortaleza Declaration)
option b, c and d are correct. • Option d is incorrect: Recently Government of
India signed a $304 million pact with AIIB for
Q.94) Ans: c power transmission network in Assam. The fund
Exp: will be utilised for the ‘Assam Intra-
• Statement 1 is correct: AIIB is a multilateral State Transmission System Enhancement Project’,
development bank focused on developing Asia, aiming to improve reliability, capacity and security
with members from all over the world, and invests of the power transmission network in the state.
in infrastructure and other productive sectors to
foster sustainable economic development, create Q.96) Ans: c
wealth and improve infrastructure connectivity. Exp:
• Membership to the bank is open to all members • Option c is correct: The Reserve Bank of India
of the Asian Development Bank or the World (RBI) releases House price index (HPI). It is a
Bank. The bank also allows non-sovereign entities quarterly produced index. The HPI is calculated on
to apply for membership provided their home base year 2010-11. The index is based on
countries are members. transaction level data received from housing
• Statement 2 is correct: In 2018, AIIB was registration authorities in ten major cities (viz.,
granted Permanent Observer status in the Ahmedabad, Bengaluru, Chennai, Delhi, Jaipur,
deliberations of both the United Nations General Kanpur, Kochi, Kolkata, Lucknow and Mumbai).
Assembly and the Economic and Social Council, "The all-India HPI contracted by (-) 0.2 per cent on
the two development-focused principal organs of a sequential basis, largely due to decline in the
the global body house prices in Delhi, Bengaluru, Ahmedabad and
Jaipur; Mumbai recorded the highest sequential
Q.95) Ans: a rise," the statement said. However, on an annual
Exp: basis), the all-India HPI increased by 3.9 per cent
• Option a is correct: During the fifth BRICS summit in the fourth quarter of 2019-20. The index had
in Durban (2013), the leaders agreed on the risen by 3 per cent in the previous quarter and 3.6
feasibility of establishing the New Development per cent In Jan-March 2019.
Bank and made the decision to do so. It was also
agreed that the initial contribution to the Bank Q.97) Ans: b
should be substantial and sufficient for it to be Exp:
effective in financing infrastructure and finally • Option b is incorrect: The Bank only finances
during the sixth BRICS Summit in Fortaleza (2014), sovereign governments directly or projects
the leaders signed the Agreement establishing the backed by sovereign governments. It provides
e

New Development Bank (NDB). loans, guarantees, advisory services, and risk
in

• Option b is incorrect: Unlike the World Bank, management products to middle-income and
nl

which assigns votes based on capital share, in the creditworthy low-income countries. It does not
l.o

New Development Bank each participant country cater to the private entities.
will be assigned one vote, and none of the • The International Bank for Reconstruction and
ria

countries will have veto power. Development (IBRD) is an international financial


e

• Option c is incorrect: “The Bank shall have an institution, established in 1944 and
at

initial authorized capital of headquartered in Washington, D.C., United


cm

US100billion.TheinitialsubscribedcapitalshallbeUS States, that is the lending arm of World Bank


100billion.TheinitialsubscribedcapitalshallbeUS 50 Group. The IBRD offers loans to middle-income
ps

billion, equally shared among founding members. developing countries. The IBRD is the first of five
member institutions that compose the World
.u

The first chair of the Board of Governors shall be


from Russia. The first chair of the Board of Bank Group. The initial mission of the IBRD in
w
w

PRAYAS TEST 24 40
w
Contact us :info@onlyias.com

OnlyIAS Nothing Else Visit :dpp.onlyias.in


Contact : +91-7007 931 912

1944, was to finance the reconstruction of projecting the income of a nation’s populace. The
European nations devastated by World War II. The Gini coefficient is the most commonly used
IBRD and its concessional lending arm, the estimation of inequality.
International Development Association (IDA), are • Statement 1 is correct: The Gini index is a
collectively known as the World Bank as they measure of the distribution of income across a
share the same leadership and staff. population. A higher Gini index indicates greater
• India is the largest IBRD client of the World Bank inequality, with high-income individuals receiving
and is a founding member of IBRD. Infrastructural much larger percentages of the total income of
development of the Indian Railways was the first the population. Practically, the value falls
project undertaken with the help of IBRD just between 0 and 1 for most countries. In countries
after independence and it became an important with high inequality, the value would be close to
source of long -term funding for developmental 1.
projects. • Statement 2 is correct: It is published by the
• IBRD has maintained a triple-A credit rating since World Bank. As per the World Bank, India’s Gini
1959. Its high credit rating allows it to borrow at Index is 35.2 (0.35) as of March 2020.
low cost and offer middle-income developing
countries access to capital on favorable terms in
larger volumes, with longer maturity period.

Q.98) Ans: a
Exp:
• Option a is correct: In 1990, the World Bank
introduced the concept of ‘poverty line’ to
capture absolute poverty. As per the revised
measures (2017), the World Bank defines extreme
poverty as someone living on less than US1.90 a
day at 2011 international prices. The
'international poverty line' is currently set at $1.90
a day at 2011 international prices.

Q.99) Ans: a
Exp:
• Option a is correct: Poverty trap is a spiraling
mechanism which forces people to remain poor. It
is so binding in itself that it doesn't allow the poor
people to escape it. Poverty trap generally
happens in developing and under-developing
countries, and is caused by a lack of capital and
credit to people. Many factors contribute to
e

creating a poverty trap, including limited access to


in

credit and capital markets, extreme


nl

environmental degradation (which depletes


l.o

agricultural production potential), corrupt


governance, capital flight, poor education
ria

systems, disease ecology, lack of public health


e

care, war, and poor infrastructure.


at
cm

Q.100) Ans: c
Exp:
ps

• In economics, the Gini coefficient, also known as


the Gini index or Gini ratio, is a measure of
.u

demographic distribution with the aim of


w
w

PRAYAS TEST 24 41
w

You might also like